You are on page 1of 44

This is Only for Sample

for Full Book Order Online and Available at All Leading Bookstores

PHYSICS

m
11th Standard

co
VOLUME - I & II

s.
ok
Based on the updated New Textbook
e
FreWorkbook
e
Practic with
anual
Lab M
o
Salient Features
ab
Prepared as per the updated New Textbook
Exhaustive Additional MCQs, VSA, SA, LA questions with answers are given in each
chapter.
ur

All the objective type (1 Mark) questions, are given with 4 options.
(i) Choose the correct option (v) Choose the correct pair
(ii) Match the following (vi) Choose the incorrect pair
(iii) Fill in the blanks (vii) Assertion-Reason
.s

(iv) Choose the odd one out (viii) Choose the correct or incorrect
statements
Govt. Model Question Paper (2018) [Govt. MQP-2018], First Mid-Term Test (2018)
w

[First Mid-2018], Quarterly Exam - 2018 [QY-2018], Half Yearly Exam - 2018
[HY-2018], March - 2019 & 2020 [Mar.-2019 & 2020], June - 2019 [Jun.-2019] Quarterly
Exam - 2019 [QY-2019], Half Yearly Exam - 2019 [HY. 2019] and September - 2020
w

[Sep. 2020] are incorporated in the appropriate sections.

Govt. Suppl. Exam September 2020 Question Paper is given.


w

SURA PUBLICATIONS
Chennai

orders@surabooks.com Ph:9600175757 / 8124201000


This is Only for Sample
for Full Book Order Online and Available at All Leading Bookstores

2021-22 Edition
© Reserved with Publishers
ISBN : 978-93-5330-061-6
Code No : SG 264

m
Author :
Mr.J.M.Joseph M.Sc.,B.Ed.,
Chennai.

co
Reviewed by :
Mr. P. George Paul M.Sc.,M.Ed., M.Phil, PGDCA
Vice-Principal
Annai Veilankanni’s Matric Hr. Sec. School

s.
81, VGP Salai, Saidapet, Chennai - 600 015

Head Office:

ok
1620, ‘J’ Block, 16th Main Road, Anna Nagar,
Chennai - 600 040.
Phones : 044-4862 9977, 044-486 27755
Mobile : 81242 01000/ 81243 01000
e-mail
o : orders@surabooks.com
website : www.surabooks.com
ab

For More Information - Contact


Queries : enquiry@surabooks.com
For Order : orders@surabooks.com
ur

Contact : 80562 94222 / 80562 15222


Whatsapp : 8124201000 / 9840926027
Online Site : www.surabooks.com
For Free Study Materials Visit http://tnkalvi.in
.s

Also available for Std. - XI


w

Guides : ✤ Sura’s Zoology Long Version (EM/ TM)


✤ Sura’s Tamil ✤ Sura’s Computer Science (EM/TM)
✤ Sura’s English ✤ Sura’s Computer Applications (EM/TM)
✤ Sura’s Mathematics (EM/TM) ✤ Sura’s Computer Technology (EM/TM)
w

✤ Sura’s Physics (EM/TM) ✤ Sura’s Commerce (EM/TM)


✤ Sura’s Chemistry (EM/TM) ✤ Sura’s Economics (EM/TM)
✤ Sura’s Bio - Botany (EM/ TM) ✤ Sura’s Accountancy (EM/TM)
w

✤ Sura’s Bio - Zoology (EM/ TM) ✤ Sura’s Business Maths (EM)


✤ Sura’s Botany Long Version (EM/ TM)

Also available 1 Mark Q & A (EM/TM),2,3 Marks (EM/TM) and 5 Marks Q & A
(EM / TM) for all Subjects.

(ii)

orders@surabooks.com Ph:9600175757 / 8124201000


This is Only for Sample
for Full Book Order Online and Available at All Leading Bookstores

Note from Publisher

m
It gives me great pride and pleasure in bringing to you Sura’s Physics guide for

co
11th Standard. It is prepared as per the New Syllabus and New Textbooks Vol. I & II.
A deep understanding of the text and exercises is rudimentary to have an insight into
the subject. The students have to carefully understand the topics and exercises.

s.
Our guide encompasses all the requirements of the students to comprehend the
text and the evaluation of the textbook.
It will be a teaching companion to teachers and a learning companion to students.

ok
 Concept Map, Must know Definitions are given in all units.
 Exhaustive Additional MCQs, VSA, SA, LA, HOTS questions with answers are
given in each units.o
 These features will help students practice and learn effectively all the sections
of the textbooks.
ab
In order to learn effectively, I advise students to learn the subject sectionwise and
practice the exercises given.
Though these salient features are available in our Guide, I cannot negate the
ur

indispensable role of the teachers in assisting the student to understand the subject
thoroughly.
I sincerely believe this guide satisfies the needs of the students and bolsters the
.s

teaching methodologies of the teachers.

I pray the almighty to bless the students for consummate success in their
examinations.
w

Subash Raj, B.E., M.S.


- Publisher
w

Sura Publications

All the Best


w

(iii)

orders@surabooks.com Ph:9600175757 / 8124201000


This is Only for Sample
for Full Book Order Online and Available at All Leading Bookstores

Contents

m
VOLUME - I

co
UNIT I : Nature of Physical World and Measurement........................................................................1 - 38

UNIT II : Kinematics.........................................................................................................................39 - 87

s.
UNIT III : Laws of motion...............................................................................................................88 - 133

UNIT IV : Work, Energy and Power...............................................................................................134 - 167

ok
UNIT V : Motion of System of Particles and Rigid Bodies.............................................................168 - 204

VOLUME - II
o
ab
UNIT VI : Gravitation....................................................................................................................205 - 235

UNIT VII : Properties of Matter.......................................................................................................236 - 270


ur

UNIT VIII : Heat and Thermodynamics...........................................................................................271 - 321

UNIT IX : Kinetic Theory of Gases.................................................................................................322 - 349


.s

UNIT X : Oscillations....................................................................................................................350 - 387

UNIT XI : Waves............................................................................................................................388 - 424


w


Govt. Supply. Exam September Question Paper ...........................................................425 - 426
w
w

(iv)

orders@surabooks.com Ph:9600175757 / 8124201000


This is Only for Sample
for Full Book Order Online and Available at All Leading Bookstores

TO ORDER WITH US

SCHOOLS and TEACHERS


We are grateful for your support and patronage to ‘SURA PUBLICATIONS’

m
Kindly prepare your order in your School letterhead and send it to us.
For Orders contact: 80562 94222 / 80562 15222

co
DIRECT DEPOSIT
A/c Name : Sura Publications A/c Name : Sura Publications
Our A/c No. : 36550290536 Our A/c No. : 21000210001240

s.
Bank Name : STATE BANK OF INDIA Bank Name : UCO BANK
Bank Branch : PADI Bank Branch : Anna Nagar West
IFSC : SBIN0005083 IFSC : UCBA0002100

ok
A/c Name : Sura Publications A/c Name : Sura Publications
Our A/c No. : 6502699356 Our A/c No. : 1154135000017684
Bank Name : INDIAN BANK Bank Name : KVB BANK
Bank Branch :
o
ASIAD COLONY Bank Branch : Anna Nagar
IFSC : IDIB000A098 IFSC : KVBL0001154
ab
After Deposit, please send challan and order to our address.
email : orders@surabooks.com / Whatsapp : 81242 01000.

DEMAND DRAFT / CHEQUE


ur

Please send Demand Draft / cheque in favour of ‘SURA PUBLICATIONS’


payable at Chennai.
.s

The Demand Draft / cheque should be sent with your order in School letterhead.

STUDENTS
w

Order via Money Order (M/O) to

Sura Publications
w

1620, ‘J’ Block, 16th Main Road, Anna Nagar,


Chennai - 600 040.
w

Phones : 044-4862 9977, 044-486 27755


Mobile : 80562 94222 / 80562 15222
email : orders@surabooks.com Website : www.surabooks.com

(v)

orders@surabooks.com Ph:9600175757 / 8124201000


This is Only for Sample
for Full Book Order Online and Available at All Leading Bookstores

PHYSICS

m
co
VOLUME - I

s.
ok
o Contents

VOLUME - I
ab

UNIT I : Nature of Physical World and Measurement........................................................................1 - 38

UNIT II : Kinematics.........................................................................................................................39 - 87
ur

UNIT III : Laws of motion...............................................................................................................88 - 133

UNIT IV : Work, Energy and Power...............................................................................................134 - 167


.s

UNIT V : Motion of System of Particles and Rigid Bodies.............................................................168 - 204


w
w
w

(vi)

orders@surabooks.com Ph:9600175757 / 8124201000


This is Only for Sample
for Full Book Order Online and Available at All Leading Bookstores

UNIT
Nature of
01 Physical World and

m
Measurement

co
Concept Map

s.
Nature of Physical World and Measurement

ok
Fundamental Quantities
and their units
Science - Introduction o Measurement

Derived Quantities and


their units
ab
Scientific Method

FPS
Physics - Introduction
ur

Measurement Systems CGS

Branches of Physics MKS


.s

Length Mass Time


SI
Scope and Excitement
w

of Physics
Errors in Measurement
w

Physics in relation with


Tech. & Society Significant Figures &
w

Rounding off

Physics relation to other Limitations


Sciences. Dimensional Analysis
Applications
[1]

orders@surabooks.com Ph:9600175757 / 8124201000


This is Only for Sample
for Full Book Order Online and Available at All Leading Bookstores
2 Sura’s  XI Std - Physics ➠ Volume - I ➠ Unit 01 ➠ Nature of Physical World and Measurement

FORMULAE TO REMEMBER
(1) Distance travelled by light in one year in vaccuum = [velocity of light × 1 year in seconds]

m
= 3 × 108× 365.25 × 24 × 60 × 60
= 9.467 × 1015m
(2) π radian = 180º

co
180º 180º × 7
(3) 1 radian = = = 57.27 º
π 22
(4) Also 1º (degree of arc) = 60′ (minute of arc) and 1′ (minute of arc) = 60′′ (seconds of arc)

s.
Relations between radian, degree and minutes:
π
(5) 1º = rad = 1.745 × 10–2 rad
180

ok
1º 1.745 × 10 −2
(6) 1′ = = = 2.908 × 10–4 rad
60 60
≈ 2.91 × 10–4 rad
o
1º 1.745 × 10 −2
(7) 1′′ = = = 4.847 × 10–6 rad
3600 3600
ab
≈ 4.85 × 10–6 rad

(8) Derived unit: unit of distance m −1


Example : unit of speed = unit of time = s = ms
ur

a1 + a2 + a3 + .......... + an 1 i=n
(9) Absolute error am = or am = n ∑ ai ; am → true value of measured quantity,
n i =1
.s

n → number of values
n
1
(10) Mean Absolute error Dam= n ∑ ∆ ai ; D am → Mean absolute error, n → number of values
w

i =1

∆am
(11) Relative error (or) Fractional error Da = ; am → Mean value
am
w

∆am
(12) Percentage error, Da = a × 100%
w

orders@surabooks.com Ph:9600175757 / 8124201000


This is Only for Sample
for Full Book Order Online and Available at All Leading Bookstores
Sura’s  XI Std - Physics ➠ Unit 01 ➠ Nature of Physical World and Measurement 3

some common practical units


(1) 1 Fermi, 1 fm = 10–15 m
(2) 1 Angstrom, 1Å = 10–10 m

m
(3) 1 nanometer, 1nm = 10–9 m
(4) 1 micron (or) micro meter, 1µm = 10–6 m
(5) 1 Light year = 9.467 × 1015m

co
(6) 1 Astronomical unit, 1 AU = 1.496 × 1011 m
(7) 1 Parallactic second, 1 parsec = 3.08 × 1016 m = 3.26 light year
(8) 1 CSL = 1.4 times, the mass of the sun

s.
(9) 1 shake = 10–8 s (or) 10 nanoseconds

Prefixes for Powers of Ten

ok
Multiple Prefix Symbol Sub multiple Prefix Symbol
101 deca da 10–1 deci d
102 hecto h 10–2 centi c
o
103 kilo k 10–3 milli m
ab
106 mega M 10–6 micro µ
109 giga G 10–9 nano n
1012 tera T 10–12 pico p
ur

1015 peta P 10–15 femto f


1018 exa E 10–18 atto a
1021 zetta Z 10–21 zepto z
.s

1024 yotta Y 10–24 yocto y


w

Important Terms & Definitions


w

Science : Science is the systematic organization of knowledge gained through


observation, experimentation and logical reasoning.
Physics : Physics is most basic science which deals with study of nature and natural
w

phenomena.
Unification : Attempting to explain diverse physical phenomena with a few concepts and
laws.
Reductionism : An attempt to explain a macroscopic system in terms of its microscopic
constituents.

orders@surabooks.com Ph:9600175757 / 8124201000


This is Only for Sample
for Full Book Order Online and Available at All Leading Bookstores
4 Sura’s  XI Std - Physics ➠ Volume - I ➠ Unit 01 ➠ Nature of Physical World and Measurement

Technology : The application of the principles of physics, i.e. knowledge for practical
purposes in various fields to invent and produce useful products or to solve
problems.
Classical mechanics : The study of forces acting on bodies whether at rest or in motion
Thermodynamics : The study of the relationship between heat and other forms of energy

m
Optics : The study of light
Electricity and : The study of electricity and magnetism and their mutual relationship
magnetism

co
Acoustics : The study of the production and propagation of sound waves
Astrophysics : The branch of physics which deals with the study of the physics of astronomical
bodies

s.
Relativity : One of the branches of theoretical physics which deals with the relationship
between space, time and energy particularly with objects moving in different
ways .

ok
Quantum mechanics : The study of the discrete nature of phenomena at the atomic and subatomic
levels
Atomic physics : The branch of physics which deals with the structure and properties of the atom
o
Nuclear physics : The branch of physics which deals with the structure, properties and reaction
of the nuclei of atoms.
ab
Condensed matter : The study of the properties of condensed materials (solids, liquids and
physics those intermediate between them and dense gas). It branches into various
sub-divisions including developing fields such as nano science, photonics
etc. It covers the basics of materials science, which aims at developing new
ur

material with better properties for promising applications.


High energy physics : The study of the nature of the particles.
Range of time scales : Astronomical scales to microscopic scales, 1018s to 10−22s.
.s

Range of masses : From heavenly bodies to electron, 1055 kg (mass of known observable
universe) to 10−31 kg (mass of an electron) [the actual mass of an electron is
9.11×10–31 Kg].
w

Measurement : The comparison of any physical quantity with its standard unit is known as
measurement.
w

Physical Quantities : Quantities that can be measured, and in terms of which, laws of physics are
described are called physical quantities.
Fundamental : Fundamental or base quantities are quantities which cannot be expressed in
w

Quantities terms of any other physical quantities. These are length, mass, time, electric
current, temperature, luminous intensity and amount of substance.
Derived Quantities : Quantities that can be expressed in terms of fundamental quantities are called
derived quantities. Eg. area, volume, velocity, acceleration, force.
Unit of the quantity : An arbitrarily chosen standard of measurement of a quantity, which is accepted
internationally is called unit of the quantity.

orders@surabooks.com Ph:9600175757 / 8124201000


This is Only for Sample
for Full Book Order Online and Available at All Leading Bookstores
Sura’s  XI Std - Physics ➠ Unit 01 ➠ Nature of Physical World and Measurement 5

Fundamental or base : The units in which the fundamental quantities are measured are called
units fundamental or base units.
Derived Unit : The units of measurement of all other physical quantities, which can be
obtained by a suitable multiplication or division of powers of fundamental
units, are called derived units.

m
System of units : A complete set of units which is used to measure all kinds of fundamental
and derived quantities is called a system of units.
Radian (rad) : One radian is the angle subtended at the centre of a circle by an arc equal in
length to the radius of the circle.

co
Steradian (sr) : One steradian is the solid angle subtended at the centre of a sphere, by that
surface of the sphere, which is equal in area, to the square of radius of the sphere
macrocosm : Large objects like the galaxy, stars, Sun, Earth, Moon etc., and their distances

s.
constitute a macrocosm. It refers to a large world, in which both objects and
distances are large.
Microcosm : Objects like molecules, atoms, proton, neutron, electron, bacteria etc., and

ok
their distances constitute microcosm, which means a small world in which
both objects and distances are small-sized.
Parallax : The shift in the position of an object (say, a pen) when viewed with two eyes,
keeping one eye closed at a time is known as Parallax.
o
Accuracy : Accuracy refers to how far we are from the true value.
Precision : Precision refers to how well we measure.
ab
Systematic errors : Systematic errors are reproducible inaccuracies that are consistently in the
same direction.
Least count error : Least count is the smallest value that can be measured by the measuring
instrument, and the error due to this measurement is least count error.
ur

Astronomical unit : It is the mean distance of the centre of sun from the centre of earth
1AU = 1.496 × 1011m
Light year : It is the distance travelled by light in vacuum in one year.
.s

1 light year = 9.467 × 1015 m.


Significant figures : The digits which tell us the number of units we are reasonably sure of having
counted in making a measurement are called significant figures.
w

Error measurement : The uncertainity in the measurement of a physical quantity is called error.
Error = True value – Measured value
w

Dimensions : Dimensions of a physical quantity are the powers to which the fundamental
quantities must be raised.
Dimensional constant : Physical quantities which possess dimensions and have constant values are
w

called dimensional constants.


Dimensionless : Quantities which have constant values and also have no dimensions are called
constant dimensionless constants.
Principle of : The principle of homogeneity of dimensions states that the dimensions of all
homogeneity of the terms in a physical expression should be the same.
Dimensions

orders@surabooks.com Ph:9600175757 / 8124201000


This is Only for Sample
for Full Book Order Online and Available at All Leading Bookstores
6 Sura’s  XI Std - Physics ➠ Volume - I ➠ Unit 01 ➠ Nature of Physical World and Measurement

EVALUATION
I. Multiple Choice Questions: 8. The dimensional formula of Planck’s constant
h is [AMU, Main, JEE, NEET]
1. One of the combinations from the fundamental (a) [ML2T–1] (b) [ML2T–3]

m
physical constants is hc . The unit of this (c) [MLT–1] (d) [ML3T–3]
expression is G
 [Ans. (a) [ML2T–1]]
2 3 –1
(a) kg (b) m (c) s (d) m 9. The velocity of a particle v at an instant t is

co
 [Ans. (a) kg2] given by n = at + bt2. The dimensions of b is
(a) [L] (b) [LT–1]
2. If the error in the measurement of radius is
2%, then the error in the determination of (c) [LT–2] [Ans. (d) [LT–3]]
(d) [LT–3]
volume of the sphere will be 10. The dimensional formula for gravitational

s.
[Sep. - 2020]
(a) 8% (b) 2% (c) 4% (d) 6%
constant G is  [Related to AIPMT 2004]
(a) [ML3T–2] (b) [M–1L3T–2]
 [Ans. (d) 6%]
(c) [M–1L–3T–2] (d) [ML–3T2]

ok
3. If the length and time period of an oscillating  [Ans. (b) [M–1L3T–2]]
pendulum have errors of 1% and 3% 11. The density of a material in CGS system of units
respectively then the error in measurement of is 4 g cm–3 . In a system of units in which unit of
acceleration due to gravity is length is 10 cm and unit of mass is 100 g, then

o
[Related to AMPMT 2008] [HY-2018] the value of density of material will be
(a) 4% (b) 5% (c) 6% (d) 7% (a) 0.04 (b) 0.4 (c) 40 (d) 400
ab
 [Ans. (d) 7%]  [Ans. (c) 40]
4. The length of a body is measured as 3.51 m, 12. If the force is proportional to square of
if the accuracy is 0.01m, then the percentage velocity, then the dimension of proportionality
constant is  [JEE–2000] [QY-2019]
error in the measurement is  (March 2020)
ur

(a) [MLT0] (b) [MLT–1]


(a) 351% (b) 1%
(c) [ML–2T] (d) [ML–1T0]
(c) 0.28% (d) 0.035%
 [Ans. (d) [ML–1T0]]
 [Ans. (c) 0.28%] 1
13. The dimension of ( µ 0 ε 0 )

2 is 
.s

[HY-2019]
5. Which of the following has the highest number  [Main AIPMT 2011]
of significant figures? (a) length (b) time
(a) 0.007 m2 (b) 2.64 × 1024 kg (c) velocity (d) force
w

(c) 0.0006032 m2 (d) 6.3200 J  [Ans. (c) velocity]


 [Ans. (d) 6.3200 J] 14. Planck’s constant (h), speed of light in vacuum
(c) and Newton’s gravitational constant (G)
w

6. If π = 3.14, then the value of π2 is


are taken as three fundamental constants.
 [QY. - 2018; Jun.-2019]
Which of the following combinations of these
(a) 9.8596 (b) 9.860 has the dimension of length? [NEET 2016
w

(c) 9.86 (d) 9.9 [Ans. (c) 9.86] (phase II)]


7. Which of the following pairs of physical
hG hG hc Gc
quantities have same dimension?  [Mar. - 2019] (a) 3
(b) 5 (c) (d) 3
(a) force and power (b) torque and energy c 2 c 2 G h2
(c) torque and power (d) force and torque hG
[Ans. (a) 3 ]
 [Ans. (b) torque and energy]  c2

orders@surabooks.com Ph:9600175757 / 8124201000


This is Only for Sample
for Full Book Order Online and Available at All Leading Bookstores
Sura’s  XI Std - Physics ➠ Unit 01 ➠ Nature of Physical World and Measurement 7

15. A length-scale (l) depends on the permittivity 3. Write the rules for determining significant
(e) of a dielectric material, Boltzmann figures.
constant (kB), the absolute temperature (T), Ans. (i) All non-zero digits are significant
the number per unit volume (n) of certain (ii) All zeros between two non-zero digits are
charged particles, and the charge (q) carried significant

m
by each of the particles. Which of the following (iii) All zeros to the right of a non-zero digit but
expression for l is dimensionally correct? to the left of a decimal point are significant.
 [JEE (advanced) 2016] (iv) a) The number without a decimal point,
the terminal or trailing zero(s) are not

co
(a) l = nq 2 (b) l = e kBT
e kBT nq2 significant.
b) All zeros are significant if they come
(c) l = q2 (d) l = q2 from a measurement
2
e nkBT (v) If the number is less than 1, the zero (s) on

s.
e n 3 kBT
εkBT the right of the decimal point but to left of
 [Ans. (b) l= ]
nq2 the first non-zero digit are not significant.
II. Short Answer Questions. (vi) All zeros to the right of a decimal point and

ok
1. Briefly explain the types of physical quantities. to the right of non-zero digit are significant.
Ans. (i) Physical quantities are classified into two types. (vii) The number of significant figures does not
There are fundamental and derived quantities. depend on the system of units used.
(ii) Fundamental or base quantities are quantities
4. What are the limitations of dimensional
o
which cannot be expressed in terms of any
other physical quantities. These are length, analysis? [Govt. MQP-2018; HY-2018; Jun.-2019]
mass, time, electric current, temperature, Ans. Limitations of Dimensional analysis:
ab
luminous intensity and amount of substance. (i) This method gives no information about the
(iii) Quantities that can be expressed in terms of dimensionless constants in the formula like
fundamental quantities are called derived 1, 2, ……..π,e, etc.
quantities. For example, area, volume, velocity, (ii) This method cannot decide whether the given
ur

acceleration, force. quantity is a vector or a scalar.


2. How will you measure the diameter of the Moon (iii) This method is not suitable to derive relations
using parallax method? [HY-2018 & 19; QY-2019] involving trigonometric, exponential and
logarithmic functions.
.s

Ans. O - observation point on earth.


(i) In diagram, O is the observation d
(iv) It cannot be applied to an equation involving
B
point on the earth and d is the A more than three physical quantities.
diameter of moon. An astronomical
w

(v) It can only check on whether a physical


telescope held at O is focussed relation is dimensionally correct but not
on moon, the image is observed S
the correctness of the relation.
into moon of a circular disc. θ
For example using dimensional analysis,
w

(ii) ∠AOB = θ s = ut + 1/3 at2 is dimensionally correct


O
S - average distance between whereas the correct relation is s = ut +1/2 at2.
moon and the surface of earth.
w

5. Define precision and accuracy. Explain with


(iii) As 'S' is very large compared to the diameter,
d of the moon, the diameter of the moon is one example.
considered as a circular arc of radius, S. Ans. Precision: The closeness of two or more
d = S × θ. measurements to each other is known as
Hence d can be calculated, when 'S' is known and precision.
θ is measured.

orders@surabooks.com Ph:9600175757 / 8124201000


This is Only for Sample
for Full Book Order Online and Available at All Leading Bookstores
8 Sura’s  XI Std - Physics ➠ Volume - I ➠ Unit 01 ➠ Nature of Physical World and Measurement

Accuracy: The closeness of a measured value to (c) Knowing the distance x, the height h can be
the actual value of the object being measured is determined.
called accuracy.
RADAR method
Example: Suppose a man's true height is exactly (i) The word RADAR stands for radio detection
5′9″. When it is measured with a yardstick, the and ranging.
value is 5′0″. Hence measurement is not accurate.

m
(ii) A radar can be used to measure accurately
When height is measured with a laser yardstick, the distance of a nearby planet such as Mars.
the value is 5′9″ then measurement is accurate. In this method, radio waves are sent from
If the height is measured consistently as 5′0″ transmitters which, after reflection from the

co
with a yardstick, then measurements are precise. planet, are detected by the receiver.
III. Long Answer Questions (iii) By measuring, the time interval (t) between the
1. (i) Explain the use of screw gauge and vernier instants the radio waves are sent and received,
caliper in measuring smaller distances. the distance of the planet can be determined
(ii) Write a note on triangulation method and radar as

s.
method to measure larger distances. [Govt. MQP- Distance(d) = Speed of radio waves × time
2018]
v×t
Ans. Measurement of small distances: taken d =
2

ok
(i) T he screw gauge is an instrument used for
measuring accurately the dimensions of objects (iv) where v is the speed of the radio wave. As
up to a maximum of about 50 mm. the time taken (t) is for the distance covered
(a) It is used for measuring external dimensions. during the forward and backward path of the
i.e. diameter.
o radio waves, it is divided by 2 to get the actual
(b) The least count of the screw gauge is distance of the object.
0.01 mm. (v) This method can also be used to determine
ab
A vernier caliper is a versatile instrument the height, at which an aeroplane flies from
for measuring the dimensions of an object the ground.
namely diameter of a hole, or a depth of a
hole. i.e. internal & external dimensions. 2. Explain in detail the various types of errors.
The least count of vernier caliper is  [Mar., QY-2019]
ur

Ans. Types of errors :


0.1 mm
(ii) For measuring larger distances such as the height of (a) Systematic error (b) Random error
a tree, distance of the Moon or a planet from the (c) Gross error
Earth, triangulation method, parallax method (a) Systematic errors : They are reproducible
.s

inaccuracies that are consistently in the same


and radar method are used. direction.
Triangulation method for the height of an
It is classified as follows :
accessible object : 
w

(March 2020)
(1) Instrumental errors : It arises when an
(a) Let AB = h be A instrument is not calibrated properly at the
the height of the time of manufacturing. It can be corrected
tree or tower to by choosing accurate instruments.
w

be measured. Let (2) Imperfections in experimental technique


C be the point h
or procedure: It is due to the limitation in
of observation at the experimental arrangement. To overcome
w

distance x from B. θ this, necessary and proper correction is


Place a range finder C x B
to be applied.
at C and measure the Triangulation method (3) Personal errors : These errors are due to
angle of elevation, individuals performing the experiment,
ÐACB = θ as shown in Figure. may be due to incorrect initial setting up
(b) From right-angled triangle ABC, tan θ = AB = h to the experiment or carelessness of the
(or) height h = x tan θ. BC x individual making the observation due
to improper precautions.

orders@surabooks.com Ph:9600175757 / 8124201000


This is Only for Sample
for Full Book Order Online and Available at All Leading Bookstores
Sura’s  XI Std - Physics ➠ Unit 01 ➠ Nature of Physical World and Measurement 9

(4) Errors due to external causes : The Measured value of B = B ± ΔB


change in the external conditions during an Consider the sum, Z = A + B
experiment can cause error in measurement. The error ΔZ in Z is then given by
For example, changes in temperature,
humidity, or pressure during measurements Z ± ΔZ = (A ± ΔA) + (B ± ΔB)
may affect the result of the measurement. = (A + B) ± (ΔA + ΔB)

m
(5) Least count error : Least count is the = Z ± (ΔA + ΔB)
smallest value that can be measured by (or) ΔZ = ΔA + ΔB
the measuring instrument, and the error The maximum possible error in the sum of two

co
due to this measurement is least count quantities is equal to the sum of the absolute
error.
errors in the individual quantities.
(b) Random error :
(ii) Error in the difference of two quantities :
(1) It arises due to random and unpredictable
variations in experimental conditions like Let ΔA and ΔB be the absolute errors in the

s.
pressure, temperature, voltage supply, two quantities, A and B, respectively. Then,
etc. Measured value of A = A ± ΔA
(2) It also arises due to personal errors by the Measured value of B = B ± ΔB

ok
observer. It is sometimes called ‘chance Consider the difference, Z = A – B
errors’. The error ΔZ in Z is then given by
(3) It can be minimised by repeating the Z ± ΔZ = (A ± ΔA) – (B ± ΔB)
observations a large number of times
= (A – B) ± ΔA + ΔB
the observations.
o
and taking the arithmetic mean of all
= Z ± ΔA + ΔB
(c) Gross error : (or) ΔZ = ΔA+ ΔB
ab
(1) The error caused due to the shear carelessness The maximum error in difference of two
of an observer is called gross error. quantities is equal to the sum of the absolute
(2) It can be minimized only when an observer errors in the individual quantities.
is careful and mentally alert. (iii) Error in the product of two quantities
ur

3. What do you mean by propagation of errors? Let ΔA and ΔB be the absolute errors in the
Explain the propagation of errors in addition two quantities, A and B, respectively. Consider
and multiplication. (March 2020)
the product Z = AB
Ans. A number of measured quantities may be involved
.s

in the final calculation of an experiment. Different = (AB) ± (A Δ B) ± (B Δ A) ± (ΔA . ΔB)


types of instruments might have been used for Dividing L.H.S by Z and R.H.S by AB, we
taking readings. Then we may have to look at the get,
w

errors in measuring various quantities, collectively. ∆Z ∆B ∆A ∆A . ∆B


The error in the final result depends on 1± = 1± ± ±
Z B A A B
(i) The errors in the individual measurements As ΔA /A, ΔB / B are both small quantities,
w

(ii) On the nature of mathematical operations ∆A ∆B


their product term . can be neglected.
performed to get the final result. So we should A B
know the rules to combine the errors. The maximum fractional error in Z is
w

These types of combination of errors are known


∆Z  ∆A ∆B 
as Propagation of errors. = ± + 
Z  A B 
(i) Error in the sum of two quantities :
The maximum fractional error in the product
Let ΔA and ΔB be the absolute errors in the
of two quantities is equal to the sum of the
two quantities A and B respectively. Then,
fractional errors in the individual quantities.
Measured value of A = A ± ΔA

orders@surabooks.com Ph:9600175757 / 8124201000


This is Only for Sample
for Full Book Order Online and Available at All Leading Bookstores
10 Sura’s  XI Std - Physics ➠ Volume - I ➠ Unit 01 ➠ Nature of Physical World and Measurement

4. Write short notes on the following. To convert a physical quantity from one system
(a) Unit of units to another
(b) Rounding - off (i) This is based on the fact that the product
(c) Dimensionless quantities of the numerical values (n) and its
Ans. (a) Unit : An arbitrarily chosen standard of corresponding unit (u) is a constant. i.e, n

m
measurement of a quantity, which is accepted [u] = constant (or) n1[u1] = n2[u2].
internationally is called unit of the quantity.
(ii) Consider a physical quantity which has
The units in which the fundamental quantities
are measured are called fundamental or base dimension ‘a’ in mass, ‘b’ in length and ‘c’ in

co
units and the units of measurement of all other time. If the fundamental units in one system
physical quantities, which can be obtained by are M1, L1 and T1 and the other system are
a suitable multiplication or division of powers M2, L2 and T2 respectively, then we can
of fundamental units are called derived units. write, n1 [M1a L1b T1c] = n2 [M2a L2b T2c]
(b) Rounding - off : The result given by a

s.
(iii) We have thus converted the numerical value
calculator has too many figures. In no case of physical quantity from one system of
should the result have more significant figures units into the other system.
than the figures involved in the data used

ok
for calculation. The result of calculation with IV. Numerical Problems.
numbers containing more than one uncertain
digit should be rounded off. 1. In a submarine equipped with sonar, the
(c) Dimensionless quantities : time delay between the generation of a pulse
and its echo after reflection from an enemy
o
(i) Physical quantities which have no
dimensions, but have variable values submarine is observed to be 80 s. If the speed
are called dimensionless variables. of sound in water is 1460 ms–1. What is the
ab
Examples are Gravitational constant, distance of enemy submarine?
Planck’s constant etc. Ans. The speed of sound in water v = 1460 ms–1
(ii) Quantities which have constant values Time taken by the pulse for to and fro :
and also have no dimensions are called
T 80s
ur

dimensionless constants. Examples are t = = = 40s


π, e, numbers, etc. 2 2
d
5. Explain the principle of homogeneity of dimensions. Formula : v =
What are its uses? Give example. t
.s

T
 [HY-2018 & 19] \d = v × 2 = 1460 × 40
Ans. Principle of homogeneity of dimensions :
It states that the dimensions of all the terms in a = 58400 m or 58.40 km.
w

physical expression should be the same.


2. The radius of the circle is 3.12 m. Calculate
For example
the area of the circle with regard to significant
v2 = u2 + 2as figures. 
w

[QY-2019]
[LT–1]2 = [LT–2]2 + 2 [LT–2] [L] Ans. Radius of the circle r = 3.12 m
[L2T–2] = [L2T–2] + 2[L2T–2] Area of the circle A = ?
This method is used to A = p r2 = 3.14 × 3.12 × 3.12 = 30.566016
w

(i) Convert a physical quantity from one


system of units to another. According to the rule of significant fig,
(ii) Check the dimensional correctness of a A = 30.6 m2 [Given data has three sig. fig.]
given physical equation.
(iii) Establish relations among various physical
quantities.

orders@surabooks.com Ph:9600175757 / 8124201000


This is Only for Sample
for Full Book Order Online and Available at All Leading Bookstores
Sura’s  XI Std - Physics ➠ Unit 01 ➠ Nature of Physical World and Measurement 11

3. Assuming that the frequency γ of a vibrating Diameter of Jupiter, d = ?


string may depend upon (i) applied force (F) (ii)
d
length (l) (iii) mass per unit length (m), prove θ =
D
1 F \ d = θ.D
that γ α using dimensional
l m = (824.7 × 109) × 35.72 × 4.85 × 10–6

m
analysis.  (related to JIPMER 2001) = 142872.677 × 103 m
Ans. γ ∝ la Fb mc = 142872.677 × 103 × 10–3 km
γ = K la Fb mc d = 1.428 × 105 km.

co
K - dimensionless constant of proportionality 5. The measurement value of length of a simple
a, b, c - powers of l, F, m pendulum is 20 cm known with 2 mm accuracy.
The time for 50 oscillations was measured to be
Dimensional Formula of F = [MLT–2] 40 s within 1s resolution. Calculate the percentage
Dimensional Formula of linear density accuracy in the determination of acceleration

s.
mass [M] 1 –1 due to gravity ‘g’ from the above measurement.
m= =
length [L] = [M L ]
Ans. The errors in both l & T are least count errors.
writing dimensions of γ = K la Fb mc

ok
[M0L0T–1] =[L]a [M1L1T–2]b [ML–1]c T = 2π l 2 2 l
g ⇒ T = 4π . g
= LaMbLb T–2b McL = [M]b+c[L]a+b–c [T]–2b
M0L0T–1 = [M]b+c [L]a+b–c [T]–2b ∴g = 4π2 l
T2
Applying the principle of homogeneity of
dimension
o The errors in both l & T are least count errors
b + c = 0 ....(1) ∆g ∆l 2 ∆T
= +
ab
a + b – c = 0 ....(2) g l T
1 Length of simple pendulum l = 20 cm
–2b = –1 or b = + accuracy Δ l = 2mm = 0.2 cm
2
1 Time for 50 oscillations T = 40 s
From  c = –b = − 1 ∴c=−
ur

2 2 resolution Δ T = 1 s
1 1 1 1 ∆g  0.2   1  0.2 2 1.2
From  a + – ( − ) = 0 a + + = 0 ∴ =   + 2   = + =
2 2 2 2 20 40 20
g 20 40
∴ a = −1
.s

Substituting the values a, b, c in  K=l Percentage error


1 1
−1 2
− ∆g 1.2
γ=K l F m 2 × 100 = × 100 = ± 6%
w

g 20
1 F
γ= . % accuracy in g = 6%.
l m
w

4. Jupiter is at a distance of 824.7 million km from Government Exam Question & Answers
the Earth. Its angular diameter is measured to
be 35.72˝. Calculate the diameter of Jupiter. I. Multiple Choice Questions :

w

1 Mark
Ans. Distance of Jupiter from the earth,
D = 824.7 × 106 km 1. A substance whose mass is 4.27 g occupies
D = 824.7 × 109 m 1. 3 cm3. The number of significant figure in
Angular diameter θ = 35.72" density is  [Govt. M.Q.P - 2018]
= 35.72 × (4.85 ×10–6 rad) (a) 1 (b) 2 (c) 3 (d) 4 
[1" = 4.85 ×10–6 rad]  [Ans. (d) 4]

orders@surabooks.com Ph:9600175757 / 8124201000


This is Only for Sample
for Full Book Order Online and Available at All Leading Bookstores
12 Sura’s  XI Std - Physics ➠ Volume - I ➠ Unit 01 ➠ Nature of Physical World and Measurement

2. Triple point of water is : [QY. - 2018] 3. What are random errors? How to minimise
(a) 273.16 k (b) 237.16 c it? [First Mid-2018]
(c) 273.16 c (d) 0 k  [Ans. (a) 273.16k] Ans. (i) Random errors may arise due to random and
3. Mass, temperature, electric current are unpredictable variations in experimental
________ [QY. - 2018] conditions like pressure, temperature, voltage
(a) fundamental quantities supply, etc. Random errors are sometimes

m
(b) scalar quantities (c) vector quantities called “chance error”.
(d) both a and b  [Ans. (d) both a and b] (ii) Random errors can be evaluated through
statistical analysis and can be reduced by
The significant figure of the number 0.003401

co
4. averaging over a large number of observations.
is: ________ [QY. - 2019] 4. Write down the number of significant figures
(a) 6 (b) 3 in the following: (i) 0.007 (ii) 400.[Govt. MQP-2018]
(c) 5 (d) 4  [Ans. (d) 4] Ans. (i) One (ii) One
5. The amplitude and time period of a simple 5. What are the advantages of SI system?

s.
pendulum bob are 0.05 m and 2 s respectively. [QY-2018]
Then the maximum velocity of the bob is : Ans. (i) It is a rational system, in which only one unit
 is used for one physical quantity.

ok
[Mar. - 2019]
a) 0.157 ms–1 b) 0.257 ms–1 (ii) It is a coherent system, which means all the
c) 0.10 ms–1 d) 0.025 ms–1 derived units can be easily obtained form basic
 [Ans. (a) 0.157 ms–1] and supplementary units.
(iii) It is a metric system which means that multiples
II. Very Short Answer Questions : and submultiples can be expressed as powers
 2 Marks
o of 10.
Check the dimensional correctness for the 6. What is fractional error?
ab
1. [QY-2018]
given equations.  [First Mid-2018] Ans. Ratio of mean absolute error to the mean value
1 (or) relative error or fractional error = mean
(a) V = u + at (b) s = ut + at absolute error / mean value.
2
(a) v = u + at Apply dimensional formula on
Ans.
III. Short Answer Questions :
ur

both sides
[LT−1] = [LT−1] + [LT−2] [T]
 3 Marks
[LT−1] = [LT−1] + [LT−1] 1. Write a note on radar method to measure larger
(Quantities of same dimension only can be distances.
.s

[First Mid-2018]
added) Ans. (i) The word RADAR stands for radio detection
Since dimensions on both sides are same, and ranging.
the given equation is dimensionally correct. (ii) A radar can be used to measure accurately
w

(b) [L] = [LT−1] [T] + [LT−2] [T2] the distance of a nearby planet such as Mars.
[L] = [LT−1 + 1] + [LT−2+2] In this method, radio waves are sent from
transmitters which, after reflection from the
[L] = [LT0] + [LT0]
w

planet, are detected by the receiver.


∴[L] = [L] + [L]
(iii) By measuring, the time interval (t) between the
Since dimensions on both sides are same, instants the radio waves are sent and received,
the given equation is dimensionally correct.
w

the distance of the planet can be determined


2. Round off to required significant figures as
 [First Mid-2018] Distance(d) = Speed of radio waves × time
(a) 3.1 + 1.780 + 2.046 (b) 12.637 – 2.42 v× t
taken d =
(c) 1.21 × 36.72 (d) 36.72 ÷ 1.2 2
Ans. (a) 6.96 (b) 10.22 (c) 44.4 (d) 31 where v is the speed of the radio wave. As
the time taken (t) is for the distance covered

orders@surabooks.com Ph:9600175757 / 8124201000


This is Only for Sample
for Full Book Order Online and Available at All Leading Bookstores
Sura’s  XI Std - Physics ➠ Unit 01 ➠ Nature of Physical World and Measurement 13

during the forward and backward path of the IV. Long Answer Questions :
radio waves, it is divided by 2 to get the actual  5 Marks
distance of the object.
(iv) This method can also be used to determine 1. The force F acting on a body moving in a circular
the height, at which an aeroplane flies from path depends on mass of the body(m) velocity(v)
the ground. and radius (r) of the circular path. Obtain the

m
2. The voltage across a wire is (100 ± 5) v and the expression for the force by dimensional analysis
current passing through it is (10 ± 0.2) A. Find method. (k = 1).  [First Mid-2018; Mar.-2019]
the resistance of the wire. [First Mid-2018] Ans. F ∝ ma vb rc ; F = k ma vb rc

co
Ans. Voltage V = (100 ± 5)V where k is a dimensionless constant of
Current I = (10 ± 0.2)A proportionality. Rewriting the above equation in
Resistance R = ? terms of dimensions and taking k = 1, we have
Then resistance R is given by Ohm's law [MLT–2] = [M]a [LT–1]b [L]c = [Ma Lb T–b Lc]

s.
V 100 [MLT–2] = [M]a [Lb+c] [T–b]
R = = = 10 W
I 10 Comparing the powers of M, L and T on both sides
∆R  ∆V ∆I  a = 1; b + c = 1 –b = –2; 2 + c = 1, b = 2
=  + 

ok
R  V I a = 1, b = 2 and c = –1
 ∆V ∆I   5 0.2  From the above equation we get F = ma νb rc
DR =  +  R=  +
 100 10 
× 10 mv 2
V I F = m1v2r−1 or F =
= (0.05 +0.02) × 10 = 0.07 × 10 = 0.7 r
The resistance R = (10 ± 0.7)W.
o 2. Obtain an expression for the time period T of a
simple pendulum. [The time period T depend
ab
3. Check the correctness of the equation
E = mc2 using dimensional analysis method. upon (i) mass m of the bob, (ii) length l of the
 [Govt. MQP-2018; Jun.-2019] pendulum and (iii) acceleration due to gravity
Ans. Consider the equation, E = mc2 g at the place where pendulum is suspended.
Apply dimensional formula on both sides Assume the constant, k = 2π].
ur

[]
ML2T–2 = [M] ∙ [LT–1]2 Ans.  [Govt. MQP-2018; QY, HY-2019]
ML2T–2 = [M] ∙ [L2T–2]
T ∝ ma lb gc ; T = k. ma lb gc
The equation is dimensionally correct.
4. Two resistances R1=(100 ± 3)Ω and R2=(150±2)Ω Here k is the dimensionless constant.
.s

are connected in series. What is their equivalent Rewriting the above equation with dimensions.
resistance ? [Govt. MQP-2018] [T1] = [Ma] [Lb] [LT−2]c
Ans. R1 = 100 ± 3 Ω ; R2 = 150 ± 2Ω
w

[M0L0T1] = [Ma Lb+c T−2c]


Equivalent resistance R = ?
Comparing the powers of M, L and T on both
Equivalent resistance R = R1 + R2
sides, a = 0, b + c =0, –2c = 1
= (100 ±3)+(150 ± 2) = (100 + 150) ± (3 + 2)
w

R = (250 ± 5) Ω Solving for a, b and c, we get a = 0, b = 1/2, and


c = −1/2
5. Find the dimensional formula of hC/G. [QY-2018]
From the above equation T = k. m0 l1/2 g−1/2
w

Ans. The dimensional formula for


1
planck’s constant h – [ML2T–1] l 2
l
T = k  =k g
c – [LT–1]  g
1
G – [M–1L2T–2] l 2 l
k  T = 2π
Experimentally k = 2π, hence =k
 g g
hc [ML2T −1 ][LT −1 ]
= = [M2]
G [M −1L3T −2 ]
orders@surabooks.com Ph:9600175757 / 8124201000
This is Only for Sample
for Full Book Order Online and Available at All Leading Bookstores
14 Sura’s  XI Std - Physics ➠ Volume - I ➠ Unit 01 ➠ Nature of Physical World and Measurement

3. In a series of successive measurements in This is based on the fact that the product of
an experiment, the readings of the period of the numerical values (n) and its corresponding
oscillation of a simple pendulum were found to unit (u) is a constant. i.e, n [u] = constant (or)
be 2.63 s, 2.56 s, 2.42 s, 2.71 s and 2.80 s. Calculate n1[u1] = n2[u2].
(i) the mean value of the period of oscillation
(ii) the absolute error in each measurement Consider a physical quantity which has

m
(iii) the mean absolute error (iv) the relative error dimension ‘a’ in mass, ‘b’ in length and ‘c’
(v) the percentage error. Express the results in in time. If the fundamental units in one system
proper form. [Govt. MQP-2018] are M1, L1 and T1 and the other system are
M2, L2 and T2, respectively, then we can write,

co
Ans. 
t1 = 2.63s, t2 = 2.56s, t3 = 2.42 s, n1 [M1a L1b T1c] = n2 [M2a L2b T2c].
We have thus converted the numerical value
t4 = 2.71s, t5 = 2.80 s of physical quantity from one system of units
t1 + t2 + t3 + t4 + t5 into the other system.

s.
(i) Tm =
5 (ii) To check the dimensional correctness of a
2.63 + 2.56 + 2.42 + 2.71 + 2.80 given physical equation.
= Let us take the equation of motion

ok
5
13.12 v = u + at
Tm = = 2.624 s
5 Apply dimensional formula on both sides 
Tm = 2.62 s (Rounded off to 2nd decimal
place) o [LT−1] = [LT−1] + [LT−2] [T]
(ii) Absolute error ΔT = Tm – t [LT−1] = [LT−1] + [LT−1]
D T1 = 2.62 – 2.63 = – 0.01 (Quantities of same dimension only can be
added)
ab
D T2 = 2.62 – 2.56 = +0.06s
D T3 = 2.62 – 2.42 = +0.20s We see that the dimensions of both sides are
same. Hence the equation is dimensionally
D T4 = 2.62 – 2.71 = – 0.09s
correct.
D T5 = 2.62 – 2.80 = – 0.18s
(iii) To establish the relation among various physical
ur

∑ ∆Tt quantities.
(iii) Mean absolute error =
n If the physical quantity Q depends upon the
0.01 + 0.06 + 0.20 + 0.09 + 0.18 quantities Q1, Q2 and Q3, i.e., Q is proportional
D Tm = +
.s

5 to Q1, Q2 and Q3.


0.54 Then Q ∝ Q1a Q2b Q3c ; Q = k Q1a Q2b Q3c
D Tm = = 0.108s = 0.11s
5
where k is a dimensionless constant. When the
w

 (Rounded off to 2nd decimal place) dimensional formula of Q, Q1, Q2 and Q3 are
(iv) Relative error: substituted, then according to the principle of
∆Tm 0.11 homogeneity, the powers of M, L, T are made
ST = = = 0.0419 = 0.04
w

Tm 2.62 equal on both sides of the equation. From this,


(v) Percentage error in T = 0.04 × 100% = 4% we get the values of a, b, c.
(vi) Time period of simple pendulum = T 1 2
w

= (2.62 ± 0.11)s s = ut + at
2
4. What are the applications of dimensional analysis. Substituting dimensions
Verify S = ut + 1/2 at2 by dimensional analysis. [L] = [LT–1][T] + [LT–2][T2]
 [Govt.MQP-2018; QY-2018 ; Sep.-2020] [L] = [L] + [L]
Ans. (i) 
To convert a physical quantity The equation is dimensionally correct.
from one system of units to another.

orders@surabooks.com Ph:9600175757 / 8124201000


This is Only for Sample
for Full Book Order Online and Available at All Leading Bookstores
Sura’s  XI Std - Physics ➠ Unit 01 ➠ Nature of Physical World and Measurement 15

5. Express 76 cm of mercury pressure in terms of M1 = 1g, M2 = 1 kg


Nm–2 using the method of dimensions.[Sep.-2020] L1 = 1cm, L2 = 1 m
Ans. Solution : T1 = 1s, T2 = 1 s
In cgs system 76 cm of mercury pressure As a =1, b = –1, and c = –2
= 76 × 13.6 × 980 dyne cm−2 Then

m
The dimensional formula of pressure P is 1 −1
 1g  1cm  1s 
−2

[ML−1T−2] P2 = 76 × 13.6 × 980      


1 kg   1 m  1s 
P1[M1a L1b T1c] = P2[M2a L2b T2c]

co
1 −1 −2
10−3 kg  10−2 m   1s 
M 
a
L 
b
 T1 
c = 76 × 13.6 × 980      
We have P2 = P1  1   1    1 kg   1 m  1s 
   
 M 2   L 2   T2 
–3 2
= 76 × 13.6 × 980 × [10 ] × 10
P2 = 1.01 × 105 Nm–2.

s.
Additional Questions

ok
I. Multiple Choice Questions : 5. What is the range of astronomical time scales
to microscopic scales?
 1 Mark (a) 1015s to 10–15s (b) 109s to 10–18s
A. Choose the best answer :
o (c) 1018 to 10–22s (d) 1011s to 10–16s 
[Ans. (c) 1018 to 10–22s]
ab
1. The word scientia is meaning to _______. 
(a) exact (b) to know 6. The law of electricity and magnetism is used to
(c) control (d) implement (a) Wireless communication
 [Ans. (b) to know] (b) Nuclear reactor (c) Steam engine
ur

2. Astronomical Scale is dealt with the _______ (d) Aeroplane[Ans. (a) Wireless communication]
Physics. 7. Match the following.
(a) Mesoscopic (b) Microscopic (1) Steam (a) Bernoulli's
.s

(c) Macrospic (d) None engine principle


 [Ans. (c) Macrospic] (2) Nuclear (b) Laws of
reactor thermodynamics
w

3. Microscopic group of Physics dealt with the (3) Production of (c) Controlled
study of _______. ultra high nuclear fission
(a) classical physics (b) statistical mechanics magnetic fields
w

(c) fluid mechanics (d) quantum physics (4) Aeroplane (d) Super conductivity
 [Ans. (d) quantum physics] (1) (2) (3) (4)
(a) b c a d
Which deals with the study of materials of an
w

4. (b) d a b c
intermediate length scale. (c) c d a b
(a) Mesoscopic physics (d) b c d a [Ans:(d) b c d a]
(b) Macroscopic physics
(c) Microscopic physics
(d) All the above [Ans. (a) Macroscopic physics]

orders@surabooks.com Ph:9600175757 / 8124201000


This is Only for Sample
for Full Book Order Online and Available at All Leading Bookstores
16 Sura’s  XI Std - Physics ➠ Volume - I ➠ Unit 01 ➠ Nature of Physical World and Measurement

8. Match the following fundamental forces with 17. The acceleration of 20 m/s2 in km/h2 is
respect to relative strengths. (a) 2.59 × 105 km/h2 (b) 1.29 × 105 km/h2
(1) Gravitational force (a) 1 (c) 2.0 × 103 km/h2 (d) 3.5× 105 km/h2
 [Ans. (a) 2.59 × 105 km/h2]
(2) Electromagnetic force (b) 10–39
10–2 18. Which device is used for measuring the mass

m
(3) Weak nuclear force (c)
(4) Strong nuclear force (d) 10–13
of atoms?
(a) Spectrograph (b) Fermi
(1) (2) (3) (4) (c) Telescope (d) Microscope

co
(a) a d b c  [Ans. (a) Spectrograph]
(b) b c d a 19. Which of the following statement is wrong?
(c) c d a b
(a) one fermi = 1015m
(d) c a b d [Ans: (b) b c d a] (b) All non-zero digits are significant.

s.
9. How many gram make 1 deca gram? (c) 1 AU = 1.496 × 1011m
(a) 10g (b) 100g (c) 1kg (d) 100kg (d) Speed is a derived unit.
 [Ans. (a) 10g]  [Ans. (a) one fermi = 1015m]

ok
10. 1 nano second is equivalent to
20. Which of the following statement is wrong?
(a) 10–6s (b) 10–3s
(a) Strain is a dimensionless quantity.
(c) 10–15s (d) 10–9s[Ans. (d) 10–9s]
o (b) Fundamental quantity is also called the base quantity.
11. Which unit is used to measure size of a nucleus? (c) force = mass × acceleration
(a) Angstrom (b) Micron (d) 1 Solar year = 1500 days.
 [Ans. (d) 1 Solar year = 1500 days]
ab
(c) Nano (d) Fermi
 [Ans. (d) Fermi] 21. Which of the following statement is true?
12. One parallactic second is, (a) Velocity is a fundamental unit.
(a) 3.08 × 1016m (b) 1.49 × 1011m (b) 1 Solar day = 24 hours. (c) 1 Shake = 104s
(d) mass is a derived unit.
ur

(c) 9.46 × 1015m (d) 1.66 × 10–27m


 [Ans. (a) 3.08 × 1016m]  [Ans. (b) 1 Solar day = 24 hours]
22. The number of significant figures in
13. How many light years make 1 parsec?
0.0006012 m is
.s

(a) 3.26 (b) 6.67 (a) 3 (b) 4


(c) 1.5 (d) 9.4 [Ans. (a) 3.26] (c) 7 (d) 5 [Ans. (b) 4]
14. How many AU makes one metre?
w

23. The number of significant figures in


(a) 3.26 × 1011AU (b) 1.496 × 1011AU 2.64 × 104 kg is
(c) 3.08 × 1016AU (d) 6.684 × 10–12AU (a) 2 (b) 4
[Ans. (d) 6.684 × 10–12AU] (c) 5 (d) 3 [Ans. (d) 3]
w


15. One lunar month is equal to __________ 24. The displacement of a particle moving
(a) 29 days (b) 27.3 days along x-axis with respect to times is given by
w

(c) 365 days (d) 30 days


x = at + bt2 – ct3. The dimensions of b are
 [Ans. (a) 29 days] (a) L0T–3 (b) L0T–3
(c) LT–2 (d) LT–3
16. What is the value of one light year in tera metre? Hint:  [Ans. (c) LT–2]
(a) 9.46 × 106Tm (b) 9.46 × 109 Tm x = at + bt2– ct3; L = aT + bT2– cT3
(c) 9.46 × 102Tm (d) 9.46 × 103 Tm ∴b = LT–2
 [Ans. (d) 9.46 × 103 Tm]

orders@surabooks.com Ph:9600175757 / 8124201000


This is Only for Sample
for Full Book Order Online and Available at All Leading Bookstores
Sura’s  XI Std - Physics ➠ Unit 01 ➠ Nature of Physical World and Measurement 17

25. If E & B respectively, represent electric field ∆P ∆M ∆V ∆P  ∆M ∆L 


and magnetics induction field, then the ratio = + ⇒ % = + 3.  %
P M V P  M L
E & B has the dimensions of
(a) angle (b) acceleration  ∆P 
  % = (5% + 3.3%) = (5 + 9) = 14%
(c) velocity (d) displacement P

m
Hint:  [Ans. (c) velocity]  ∆x 
30. The fractional error  
x
∆a  ∆a 
(a) ±   (b)
→ → → →
F = q (E + v × B) ±n  
 a   a 

co

 ∆a  ∆a
∴The dimensions of E are the (c) ± n log e   (d) ± n log10
→  a  a
same as those of vB
∴The dimensions of E/B = dimension of v.  [Ans. (b) ± n  ∆a  ]

s.
 a 
26. If force •F•, velocity •v•and time •T• are taken
as to fundamental units then the dimensions of 31. How many light years make 1 par sec?
(a) 3.26 LY (b) 6.67 LY

ok
mass are
(a) Fv–1T (b) Fv–1T (c) 1.5 LY (d) 9.4 LY
(c) FvT–1 (d) FvT–2  [Ans. (a) 3.26 LY]
Hint:  [Ans. (c) FvT–1] 32. If p = 3.14, then the value of p2 is
mv mv
o (a) 9.8596 (b) 9.860 (c) 9.86 (d) 9.9
F= t [m] = [F
T]  v  [Ans. (c) 9.86]
∵ a = t 
ab
t
33. Which of the following pairs of physical
27. The dimensions of K.E. is
quantities have same dimension?
(a) M2L2T–1 (b) M1L1T1 (a) Force and Power
(c) M1L2T–2 (d) M2L2T–2 (b) Stress and Pressure
ur

 [Ans. (c) M1L2T–2] (c) Momentum and Moment of force


(d) Torque and impulse of force
28. The dimensions of universal gravitational
 [Ans. (b) Stress and Pressure]
constant is
.s

(a) M–2L3T–2 (b) M–2L2T–1 34. The Dimensional formula for Boltzmann
(c) M–1L3T–2 (d) ML2T–1 constant is
(a) [ML2T–1] (b) [ATmol–1]
 [Ans. (c) M–1L3T–2]
w

(c) [ML2T–2K–1] (d) None of the above


29. The density of a cube is measured by measuring  [Ans. (c) [ML2T–2K–1]]
its mass and length of its side. If the maximum 35. Specific gravity (Relative Density) is an
w

error in the measurement of mass ansd length example for


are 5% and 3% respectively, the maxmimum (a) Dimensional Variables
error in the measurement of density is (b) Dimensionless Variables
w

(a) 9% (b) 8% (c) Dimensional Constant


(c) 14% (d) 2% (d) Dimension less Constant
 [Ans. (b) Dimensionless Variables]
Hint:  [Ans. (c) 14%] 36. 8.250 can be Rounded off to
M M (a) 8.3 (b) 8.2 (c) 8.25 (d) 8.26
Density ρ = = 3 ∆M = 5%; ∆L = 3%
V L  [Ans. (b) 8.2]

orders@surabooks.com Ph:9600175757 / 8124201000


This is Only for Sample
for Full Book Order Online and Available at All Leading Bookstores
18 Sura’s  XI Std - Physics ➠ Volume - I ➠ Unit 01 ➠ Nature of Physical World and Measurement

37. If E and B respectively, represent electric field 4. The temperature at which Saturated vapur,
and magnetic field of Induction, then the ratio pure and melting ice are all in equilibrium is
of E and B has the dimensional formula of called __________.
(a) [LT–2] (b) [MLT–2] (a) sublimation (b) melting point
(c) [LT–1] [Ans. (c)
(d) [MLT–1] [LT–1]] (c) Triple point of water (d) heat capacity

m
38. Which one has more significant figures  [Ans. (c) Triple point of water]
(a) 600800 (b) 5213.0
5. The expression for Solid Angle is __________.
(c) 2.65 × 1024 (d) 0.0006032
(a) rod/s (b) surface area / (radius)2

co
 [Ans. (b) 5213.0]
(c) (radius)2 (d) surface area / radius
39. Angle of 1 Second of arc is
 [Ans. (b) Surface Area / (radius)2]
(a) 48.5 × 10–6 rad (b) 0.485 × 10 –5 rad
(c) 4.85 × 10–6 rad (d) 48500 × 10–6 rad 6. J Kg–1K–1 is the unit for _________.

s.
 [Ans. (c) 4.85 × 10–6 rad] (a) Heat capacity (b) Latent heat
40. 1 Yotta = ______. (c) Specific heat (d) Energy
(a) 1021 (b) 10–24 (c) 10–21 (d) 1024  [Ans. (c) Specific heat]

ok
 [Ans. (d) 1024] 7. 1 degree = __________ rad.
41. If mass of an electron is 9.11 × 10–31 Kg, then (a) 1.754 × 10–2 (b) 1.745 × 102
how many electrons would weight in 1 mg? (c) 1.745 × 10–2 (d) 1.547 × 10–2
(a) 1.68 × 1018 (b) 1.097 × 1024  [Ans. (c) 1.745 × 10–2]
(c) 1.45 × 1022
o
(d) 1.970 × 1023
8. ________ means a large world in which both
 [Ans. (b) 1.097 × 1024]
objects and distances are large – sized.
ab
B. Fill in the blanks : (a) Macrocosm (b) Microcosm
1. An attempt to explain a Macroscopic system (c) Astronomy (d) Universe
in terms of its Microscopic constituents is  [Ans. (a) Macrocosm]
ur

_________. 9. The largest practical unit of mass is __________.


(a) unification (b) Reductionism (a) CSL (b) Par sec
(c) Microphysics (d) Macrophysics (c) Ly (d) AU
 [Ans. (b) Reductionism]  [Ans. (a) ChandraSekhar Limit (CSL)]
.s

2. The range of masses from heavenly bodies to 10. The error caused due to the shear carelessness
electro is ___________. of an observer is called ___________.
w

(a) 1052 kg to 10–28 kg (b) 1055 kg to 10+28 kg (a) Absolute Error (b) Gross Error
(c) 1055 kg to 10–31 kg
(d) 10–55 kg to 1031 kg (c) Instrumental Error (d) Zero Error
  [Ans. (b) Gross Error]
[Ans. (c) 1055 kg to 10–31 kg]
w

11. Quantities which have constant values and also


3. The CGS, MKS and SI system of units are
have no dimensions are called _________.
_______ system of units.
(a) Dimensionless Constants
w

(a) metric (b) cubic


(b) Dimensional variables
(c) periodic (d) atomic
(c) Dimensionless constants
 [Ans. (a) metric]
(d) Derived quantities
 [Ans. (a) Dimensionless Constants]

orders@surabooks.com Ph:9600175757 / 8124201000


This is Only for Sample
for Full Book Order Online and Available at All Leading Bookstores
Sura’s  XI Std - Physics ➠ Unit 01 ➠ Nature of Physical World and Measurement 19

12. Dimensional formula for Magnetic Induction is 3. Prefix (Symbol) Sub-Multiple


____________. (1) Zepto (z) (a) 10–18
(a) MT2A–1 (b) MT2A (2) Pico (p) (b) 10–1
(c) MT–2A–1 (d) MA–1 (3) atto (a) (c) 10–12
 [Ans. (c) MT–2A–1] (4) deci (d) (d) 10–21

m
13. Formula (or) expression for surface energy is (1) (2) (3) (4)
__________. (a) b d a c
(a) work / length (b) force / length (b) d b a c

co
(c) work / time (d) work / area (c) c d a b
 [Ans. (d) work / area] (d) d c a b [Ans:(d) d c a b]
14. Relative error is also called as _________. 4. Devices Principles
(a) Gross Error (b) Percentage Error (1) Steam engine (a) Bernollis Principle

s.
(c) Absolute Error (d) Fractional Error (2) Nuclear Reactor (b) Laws of
 [Ans. (d) Fractional error] thermodynamics
(3) Production (c) Controlled chain
15. The name Physics was introduced by
of ultra high reaction

ok
__________ in 350 B.C magnetic fields
(a) Thalus (b) Ptolemy (4) Aeroplane (d) Super
(c) Aristotle (d) Copernicus conductivity
 [Ans. (c) Aristotle] (1) (2) (3) (4)
(a) b c a d
C. Match the following :
o (b) d a b c
1. Branch Major Focus (c) c d a b
ab
(1) Acoustics (a) About space, time (d) b c d a [Ans:(d) b c d a]
and energy
(2) High energy (b) About sound 5. Types of fundamental Strengths
physics forces
(3) Quantum (c) About nature of par-
ur

(1) Gravitational force (a) 1


mechanics ticles
(4) Relativity (d) About discrete nature (2) Electro magnetic force (b) 10–39
of phenomena at (3) Weak nuclear force (c) 10–2
atomic and
(4) Strong nuclear force (d) 10–13
.s

sub-atomic levels
(1) (2) (3) (4) (1) (2) (3) (4)
(a) c b d a (a) a d b c
w

(b) c a d b (b) b c d a
(c) b c d a (c) c d a b
(d) d c b a [Ans:(c) b c d a] (d) c a b d [Ans: (b) b c d a]
w

2. Physical Quantity Unit 6. Classification of Examples


(1) Force Constant (a) Ns quantities
(2) Boltzmann Constant (b) Nm (1) Dimensional (a) Pi (T)
w

(3) Impulse (c) JK–1 variables


(4) Torque (d) Nm–1
(2) Dimension less (b) Planck's
(1) (2) (3) (4) Constant
variable
(a) d c a b
(3) Dimensional constant (c) Velocity
(b) b a c d
(c) b d a c (4) Dimension less constant (d) Strain
(d) d b a c [Ans:(a) d c a b]

orders@surabooks.com Ph:9600175757 / 8124201000


This is Only for Sample
for Full Book Order Online and Available at All Leading Bookstores
20 Sura’s  XI Std - Physics ➠ Volume - I ➠ Unit 01 ➠ Nature of Physical World and Measurement

(1) (2) (3) (4) 2. (a) Absolute Error (b) Relative Error
(a) b d a c (c) Percentage Error (d) Gross Error
(b) b c a d  [Ans. (d) Gross Error]
(c) c d b a
3. (a) Solar clock (b) Electronic Oscillators
(d) a c d b [Ans: (c) c d b a]
(c) Radio active dating (d) Electronic balance

m
7. Physical quantity Dimensional  [Ans. (d) Electronic balance]
Formula
(1) Surface Tension (a) [ML–1T–1] 4. (a) Energy (b) Work

co
(2) Heat Capacity (b) [ML2] (c) Torque (d) Force [Ans. (d) Force]
(3) Moment of Inertia (c) [MT–2]
5. (a) Length (b) Mass
(4) Co-efficient of (d) [ML2T–2K–1]
(c) Time (d) Volume [Ans. (d) Volume]
viscosity
(1) (2) (3) (4) (a) f.p.s (b) c.g.s

s.
6.
(a) c d b a (c) m.k.s (d) r.m.s [Ans. (d) r.m.s]
(b) a d c b
7. (a) Optics (b) Acoustics

ok
(c) a c d b
(c) Astrophysics (d) Nuclear Physics
(d) b a c d [Ans: (a) c d b a]
 [Ans. (d) Nuclear Physics]
8. Errors Cause
(1) Systematic (a) due to shear 8. (a) Force constant (b) Planck's constant
(c) Boltzmann constant (d) Refractive Index
(2)
Errors
Random Errors (b)
o
carelessness
Fractional Error  [Ans. (d) Refractive Index]
E. Choose the Incorrect Pair :
ab
(3) Gross Errors (c) Chance Error
(4) Relative Errors (d) Reproducible
inaccuracies 1. (a) Work - Energy
(1) (2) (3) (4) (b) Stress - Pressure
(a) b c d a (c) Force - Tension
ur

(d) Surface Tension - Force


(b) d c a b
(c) a c d b  [Ans. (d) Surface Tension - Force]
(d) b a d c [Ans:(b) d c a b] 2. (a) Velocity - Angular velocity
.s

9. Numbers Significant figures (b) Force - Torque


(1) 40.00 (a) 6 (c) Mass - Moment of Inertia
(2) 0.030400 (b) 3 (d) Frequency - Wavelength
w

(3) 0.00345 (c) 5  [Ans. (d) Frequency - Wavelength]


(4) 307000. (d) 4 3. (a) Density - Relative Density
w

(1) (2) (3) (4) (b) Strain - refractive Index


(a) b d c a (c) π - e
(b) b c d a (d) Planck's Constant - Stefan's constant
w

(c) c d a b  [Ans. (a) Density - Relative Density]


(d) d c b a [Ans:(d) d c b a]
4. (a) Heat - Energy
D. Choose the odd one out: (b) Mass - Inertia
(c) Charge - Current
1. (a) Specific gravity (b) Strain
(d) Moment of force - Torque
(c) refractive index (d) Planck's constant
 [Ans. (c) Charge - Current]
 [Ans. (d) Planck's constant]

orders@surabooks.com Ph:9600175757 / 8124201000


This is Only for Sample
for Full Book Order Online and Available at All Leading Bookstores
Sura’s  XI Std - Physics ➠ Unit 01 ➠ Nature of Physical World and Measurement 21

F. Choose the Correct Pair : 4. Assertion: In centimeter, the spelling meter


is internationally accepted.
1. (a) 30.00 - 2009 (b) 0.00345 - 2.6
Reason : 'Metre' is internationally used unit
(c) 0.040500 - 20100m (d) 153 - 3072
whereas 'Meter' is used by Americans.
 [Ans. (a) 30.00 - 2009] [ Ans. (d) Assertion is false but Reason is true]
Assertion: Mass due to rotational motion is

m
2. (a) Telescope - Microscope 5.
(b) Screw gauge - Radar Method moment of Inertia.
(c) Parallax Method -Vernier Caliper Reason : Rotational mass explains about
(d) Spring balance - Common balance radius of Gyration.

co
[Ans. (d) Spring balance - Common balance] [Ans. (a) Assertion and Reason are correct
and Reason is correct explanation of
3. (a) Torque - Nm
Assertion]
(b) Planck's constant - J / s
(c) Specific heat - J kg k–1
6. Assertion : Very large distances such as distance

s.
(d) Moment of Inertia - kg/m2 of a planet or star can be measured
 [Ans. (a) Torque - Nm] by parallax method.
Reason : For measuring small masses of atomic /
G. Assertion & Reason :

ok
sub - atomic particles, mass spectrograph
Directions : is used.
[Ans. (b) Assertion and Reason are true but
(a) Assertion and Reason are correct and Reason is
correct explanation of Assertion Reason is the false explanation of
 the Assertion]
o
(b) Assertion and Reason are true but Reason is the
7. Assertion :  The least value that can be measured
false explanation of the Assertion
using screw gauges, vernier calipers
ab
(c) Assertion is true but Reason is false
is called least count.
(d) Assertion is false but Reason is true
Reason : The magnitude of difference between
1. Assertion: Attempting to explain diverse the true value and the measured value
physical phenomena with a few is called relative error or fractional
ur

concepts and laws is unification. error.


Reason : Attempting to explain a macroscopic [ Ans. (c) Assertion is true but Reason is false]
system in terms of its microscopic 8. Assertion:  The rounding off of 27.653 upto 3
digits is 27.6
.s

constituents is reductionism.
[Ans. (b) Assertion and Reason are true but Reason :  10200 has three significant figures
Reason is the false explanation of [Ans. (d) Assertion is false but Reason is true]
9. Assertion : Dimensional analysis method is used
w

 the Assertion]
2. Assertion: Study of light is called optics to convert a physical quantity from
Reason : Properties of light is studied in optics. one system of units to another
Reason : 1 mv2 = mgh
w

They are Reflection, Retraction etc.,


[Ans. (a) Assertion and Reason are correct 2
[Ans. (b) Assertion and Reason are true but
and Reason is correct explanation of
Reason is the false explanation of
w

Assertion]
 the Assertion]
3. Assertion: Quantities that can be expressed in
terms of fundamental quantities
are derived quantities.
Reason : Examples are Mass, Length, Time
etc.,
[Ans. (c) Assertion is true but Reason is false]

orders@surabooks.com Ph:9600175757 / 8124201000


This is Only for Sample
for Full Book Order Online and Available at All Leading Bookstores
22 Sura’s  XI Std - Physics ➠ Volume - I ➠ Unit 01 ➠ Nature of Physical World and Measurement

h. Choose the Correct or Incorrect 7. (I)  Least count of screw gauge is 0.01mm
Statements : (II) Least count of vernier caliper is 0.1mm
Which one is correct?
1. (I)  RADAR method is used for measurement
(a) I only (b) II only
of length in the case of long distances
(c) both are correct (d) none
(II) The uncertainty in a measurement is called

m
error.  [Ans. (c) Both are correct]
Which statement is correct?
8. (I)  Parallax angle, θ = Unknown distance (x)
(a) I only (b) II only base (b)
(II) Distance of the planet,

co
(c) Both are correct (d) None e
d = Velocity of radio wave (V ) × time taken (t)
 [Ans. (c) Both are correct]
Which statement is incorrect?
2. (I)  GCGS = 6.6 × 10–8 dyne Cm2 g–2
(a) I only (b) II only
(II) T = 2 π g (c) Both are correct (d) None

s.
l  [Ans. (a) I only]
Which statement is correct? 9. (I) Frequency and angular velocity has same
(a) I only (b) II only
dimensional formula

ok
(c) Both are correct (d) None
(II) Torque is also called as rotational force
 [Ans. (a) I only]
Which one is correct?
3. (I)  Expression for charge is current / time (a) I only (b) II only
(II) Expression for Faraday constant is Avagadro (c) Both are correct (d) None
constant × elementary charge  [Ans. (c) Both are correct]
Which statement is correct?
o
(a) I only (b) II only II. Very Short Answer Questions :
ab
(c) Both are correct (d) None
 [Ans. (b) II only]  2 Marks
4. (I)  Force constant and Faraday constant are 1. What is science?
examples for Dimensional constant Ans. (i) The word science comes from a Latin word
'scientia' meaning 'to know'.
ur

(II) Radius of gyration does not depend on


moment of Inertia. (ii) Science is a systematic attempt to understand
Which statement is incorrect? natural phenomena in much knowledge gained
(a) I only (b) II only through systematic observation and experiment.
.s

(c) Both are correct (d) None 2. What are the steps involved in scientific method?
 [Ans. (b) II only] (or) What are the general features of scientific
5. (I)  The ratio of mean absolute error to the method?
w

mean value is called fractional error Ans. (i) Systematic observation


(II) Due to the wrong observations Recording,
(ii) Controlled experimentation
Random errors occur.
(iii) Reasoning (qualitative and quantitative)
w

Which statement is correct?


(a) I only (b) II only (iv) Modelling (Mathematical)
(c) Both are correct (d) None (v) Prediction and verification (theories)
 [Ans. (a) I only] 3. What is Physics?
w

6. (I)  Distance of moon from earth is 1011m Ans. (i) Physics is a branch of science.
(II) Mass of a cell is 10–10 kg (ii) The word comes from a Greek word meaning
Which statement is incorrect? 'nature'.
(a) I only (b) II only (iii) It deals with the study of nature and natural
(c) Both are correct (d) None
phenomena.
 [Ans. (a) I only]

orders@surabooks.com Ph:9600175757 / 8124201000


This is Only for Sample
for Full Book Order Online and Available at All Leading Bookstores
Sura’s  XI Std - Physics ➠ Unit 01 ➠ Nature of Physical World and Measurement 23

4. What is mechanics? 12. What is the importance of physical quantity?


Ans. (i) Mechanics is a branch of physics. What are its types?
(ii) It is divided into statics and Dynamics. Ans. (i) Physical quantities are important to
(iii) Mechanics deals with the study of motion understand the properties of materials.
of particles, deformable bodies and general (ii) It is classified into fundamental physical

m
system of particles. quantity and derived physical quantity.
5. Write a note on scope of physics. 13. Define unit of a physical quantity.
Ans. (i) Physics has a large scope as it covers a various Ans. Unit of a physical quantity is defined as the

co
physical quantities. (Length, mass, time, energy). established standard used for comparison of
(ii) It deals with the Macroscopic group (Mechanics, the given physical quantity. It is classified in to
electrodynamics, thermodynamics and optics) fundamental and derived unit.
and microscopic group (Quantum physics).
14. What are the things needed to express the

s.
Example:
measurement of a physical quantity?
Range of time scale (astronomical scale to
Ans. (i) The unit in which the quantity is measured
microscopic group (Quantum physics).
(ii) The numerical value or the magnitude of

ok
6. What is MKS system ?
the quantity (n), the number of times that
Ans. The mks system is based on metre, kilogram and
unit (u) is contained, in the given physical
second as the fundamental units of length, mass, quantity. Q = nu.
and time respectively. o
7. What is the aim of our Science Education? 15. Name the prefixes for powers of ten with its
symbol.
Ans. According to part IV Article 51A (h) of Indian
ab
Constitution “It shall be the duty of every citizen Ans. (i) 101 = deca and its symbol is da
of India to develop scientific temper, humanism (ii) 106 = mega and its symbol is M
and spirit of inquiry and reform’’. This is the aim (iii) 1012 = tera and its symbol is T
of our Science Education. (iv) 10–1 = deci and its symbol is d
ur

8. Name three practical units to measure mass. (v) 10–6 = micro and its symbol is m
Ans. (i) Pound 1b = 0.4536 kg (vi) 10–12 = pico and its symbol is p
(ii) Quintal 1q = 100–9 kg 16. Name four units to measure extremely small
.s

(iii) Atomic mass unit (1amu) = 1.66 × 10–27 kg distances.


9. Define Solar Year. Ans. Units used to measure extremely small distances
Ans. (i) It is the time taken by the earth to complete are
w

one revolution around the sun in its orbit. (i) 1 micron or micrometer, 1 mm = 10–6m
(ii) 1 Solar year = 365.25 average Solar days (ii) 1 nanometer, 1 nm = 10–9m
(iii) 1 Angstrom unit 1Å = 10–10m
w

= 366.25 Sedrial days.


10. What is Leap year? (iv) 1 Fermi, 1 Fm = 10–15m.
Ans. The year which is divisible by 4 and in which 17. Name three units to measure extremely large
w

the month of February has 29 days is called Leap distances.


year. Ans. Units used to measure extremely large distances
11. Name three practical units to measure Area. are,
Ans. (i) Barn, 1 barn = 10–28 m2 (i) Astronomical Unit :
(ii) Acre, 1 acre = 4047 m2 It is the mean distances of the earth from the
(iii) Hectare, 1 hectare = 104 m2. sun. 1 AU = 1.496 × 1011m.

orders@surabooks.com Ph:9600175757 / 8124201000


This is Only for Sample
for Full Book Order Online and Available at All Leading Bookstores
24 Sura’s  XI Std - Physics ➠ Volume - I ➠ Unit 01 ➠ Nature of Physical World and Measurement

(ii) Light year : 24. What is meant by the dimensions of a physical


It is the distance travelled by light in vacuum quantity?
in one year. 1 ly = 9.46 × 1015m Ans. The dimensions of a physical quantity are the
(iii) Parallactic second : powers to which the units of base quantities are
It is the distance at which an arc of length 1 raised to represent a derived unit of that quantity.

m
astronomical unit subtends an angle of 1 second 25. What are Dimensional variables? Give example.
of arc. 1 par sec = 3.084 × 1016m = 3.26 ly.
Ans. Physical quantities, which possess dimensions
18. What is an error? Name the three Errors in and have variable values are called dimensional

co
Measurement. variables. Examples are length, velocity, and
Ans. The uncertainty in a measurement is called an acceleration etc.
error. 26. What is meant by Dimensionless variables?
The three possible errors are Give example.

s.
(i) Systematic errors Ans. Physical quantities which have no dimensions,
(ii) Random errors but have variable values are called dimensionless
(iii) Gross errors variables. Examples are specific gravity, strain,

ok
refractive index etc.
19. What is Absolute Error.
27. Define Dimensional Constant. Give example.
Ans. The magnitude of difference between true value
and the measured value of a quantity is called Ans. Physical quantities which possess dimensions
absolute error.
o and have constant values are called dimensional
constants. Examples are Gravitational constant,
∆an = am – an
Planck’s constant etc.
20. What is Mean Absolute error?
ab
28. What is meant by Scientific method?
Ans. The arithmetic mean of the magnitude of absolute
Ans. The scientific method is a step-by-step approach
errors in all the measurements is called the mean in studying natural phenomena and establishing
absolute error. laws which govern these phenomena.
If am is the true value and Δam is the mean absolute
ur

error, then the magnitude of the quantity may lie 29. What do you mean by unification and reductionism?
between am + Δam and am - Δam. Ans. Unification: Attempting to explain diverse physical
phenomena with a few concepts and laws is unification.
21. What is Relative error?
Reductionism: An attempt to explain a macroscopic
.s

Ans. The ratio of the mean absolute error to the mean


system in terms of its microscopic constituents is
value is called relative error. This is also called reductionism.
as fractional error.
30. What is Classical mechanics?
w

Mean absolute error ∆am Ans. The study of forces acting on bodies whether at
Relative error = = a
Mean value m rest or in motion.
22. What is Percentage error? 31. What is Thermodynamics?
w

Ans. The relative error expressed as a percentage is Ans. The study of the relationship between heat and
called Percentage error. other forms of energy.
w

∆am 32. What is the meaning of Acoustics?


Percentage error = a × 100% Ans. The study of the production and propagation of
m

23. What is significant figures? sound waves.


Ans. The digits that are known reliably plus the first 33. What is Astrophysics?
uncertain digit are known as significant figures Ans. The branch of physics which deals with the study
or significant digits. of the physics of astronomical bodies.

orders@surabooks.com Ph:9600175757 / 8124201000


This is Only for Sample
for Full Book Order Online and Available at All Leading Bookstores
Sura’s  XI Std - Physics ➠ Unit 01 ➠ Nature of Physical World and Measurement 25

34. What is meant by Quantum mechanics? 42. Define the SI unit of mass. (or) What is one
Ans. The study of the discrete nature of phenomena at kilogram in SI system of units?
the atomic and subatomic levels. Ans. Kilogram is the SI unit of mass. One kilogram
35. Which branches of physics deal at the level of is the mass of the prototype cylinder of platinum
atom & nucleus? iridium alloy (whose height is equal to its diameter),

m
Ans. Atom : Atomic physics. preserved at the International Bureau of Weights
Nucleus : Nuclear physics. and Measures at Serves, near Paris, France.
36. What are types of discoveries in physics? 43. Define the SI unit of time. (or) What is one

co
Ans. (i) Accidental discoveries and well-analysed second in SI system of units?
research outcome in the laboratory based Ans. Second is the SI unit time. One second is the
on intuitive thinking and prediction. duration of 9,192,631,770 periods of radiation
(ii) For example, magnetism was accidentally corresponding to the transition between the two
observed but the reason for this strange

s.
hyperfine levels of the ground state of Cesium-133
behavior of magnets was later analysed
atom.
theoretically.
(iii) This analysis revealed the underlying phenomena 44. Define the SI unit of electric current. (or) What

ok
of magnetism. With this knowledge, artificial is one second in SI system of units? (or) Define
magnets were prepared in the laboratories. one ampere (S.I standard for current)
37. What is meant by Range of time scales? Ans. "ampere" is the SI unit of electric current. One ampere
Ans. Range of time scales: astronomical scales to is the constant current, which when maintained
microscopic scales, 1018s to 10−22s.
o in each of the two straight parallel conductors of
infinite length and negligible cross section, held
38. What is meant by Range of masses?
ab
one metre apart in vacuum shall produce a force
Ans. Range of masses: from heavenly bodies to electron,
per unit length of 2 × 10−7 N/m between them.
1055 kg (mass of known observable universe) to
10−31 kg (mass of an electron) [the actual mass 45. What is the SI unit of temperature and define
of an electron is 9.11×10–31 kg]. it? What is one kelvin in SI system of units?
ur

39. How Physics is related to technology and define Ans. Kelvin is the SI unit of temperature. One degree
technology with respect to Physics.  1 
Ans. Technology is the application of the principles of kelvin is the fraction of   of the thermodynamic
273.16 
.s

physics for practical purposes. The application of


knowledge for practical purposes in various fields temperature of the triple point of the water.
to invent and produce useful products or to solve 46. What is the SI unit of amount of substance?
w

problems is known as technology. (or) What is one mole in SI system of units?


40. In what ways physics is in relation to astronomy? (or) Define one mole (S.I standard for amount
Ans. Astronomical telescopes are used to study the motion of substance)
w

of planets and other heavenly bodies in the sky. Ans. Mole is the SI unit of amount of substance. One
Radio telescopes have enabled the astronomers to mole is the amount of substance which contains
observe distant points of the universe. Studies of as many elementary entities as there are atoms
w

the universe are done using physical principles.


in 0.012 kg of pure carbon-12.
41. Define the SI unit of length.
Ans. Metre is the SI unit of length. One metre is the
length of the path travelled by light in vacuum in
1
299, 792, 458 of a second.

orders@surabooks.com Ph:9600175757 / 8124201000


This is Only for Sample
for Full Book Order Online and Available at All Leading Bookstores
26 Sura’s  XI Std - Physics ➠ Volume - I ➠ Unit 01 ➠ Nature of Physical World and Measurement

47. What is meant by one candela? and which base 55. Define mass.
quantity is measured by this unit? (or) Define one Ans. Mass of a body is defined as the quantity of matter
candela (S.I standard for Luminous intensity) contained in a body. The SI unit of mass is kilogram
Ans. One candela is the luminous intensity in a given (kg).
direction, of a source that emits monochromatic 56. Write the masses of tiny as well as huge matter?

m
radiation of frequency 5.4 × 1014 Hz and that has Ans. A tiny mass is of electron (9.11×10−31kg). The huge
1
a radiant intensity of watt/steradian in that mass is of the known universe (= 1055 kg).
683
direction. 57. Write the methods to determine the masses of

co
48. What is meant by the triple point of water? objects?
Ans. Triple point of water is the temperature at which Ans. The mass of an object is determined in kilograms
saturated vapour, pure and melting ice are all in using a common balance. Spring balance, electronic
equilibrium. The triple point temperature of water balance are also used. For measuring larger masses

s.
is 273.16K. like that of planets, stars etc., gravitational methods
are used. For measurement of small masses of atomic/
49. What is meant by Parallax? subatomic particles etc., mass spectrograph is used.

ok
Ans. Parallax is the name given to the apparent change
58. What is clock? Write the principle and its types.
in the position of an object with respect to the
background, when the object is seen from two Ans. A clock is used to measure the time interval. An atomic
different positions. standard of time, is based on the periodic vibration
produced in a Cesium atom. Some of the clocks
50. What is 1 Light year?
o developed later are electric oscillators, electronic
Ans. 1 Light year is distance travelled by light in vacuum oscillators, solar clock, quartz crystal clock, atomic
ab
in one year. 1 Light Year = 9.467 × 1015 m. clock, decay of elementary particles, radioactive dating.
51. Define a Astronomical Unit. 59. Which units are used to measure large distance
Ans. 1 astronomical unit is the mean distance of the Earth i.e. distance of planets and stars? Which method
from the Sun. 1 AU = 1.496 × 1011 m. is used for measurement?
ur

52. What is parsec? (or) Define one parsec (parallactic Ans. AU - The mean distance between earth and star
second) 1 AU = 1.496 × 1011 m
Ans. 1 parsec, Parallactic second is the distance at which Light year - The distance travelled by light in
.s

an arc of length 1 AU subtends an angle of 1 second vacuum in one year


of arc 1 parsec = 3.08 × 1016 m = 3.26 light year. 1 light year = 9.46 × 1015 m
53. Why is the cylinder used in defining kilogram Parsec - Distance at which an arc of length
w

made up of platinum-iridium alloy? of 1 AU subtends an angle of 1 sec


Ans. This is because the platinum-iridium alloy is least at a point parsec = 3.08 × 1016 m.
affected by environment and time. Parallax method is used to measure large distance.
w

54. Write the largest and the smallest practical 60. Is it possible to have length and velocity both
unit of mass and time respectively. (or) Define as fundamental quantities? Why?
Ans. No, length is fundamental quantity whereas velocity
w

Chandrasekar Limit (CSL)


is the derived quantity.
Ans. Chandrasekhar Limit (CSL) is the largest practical
unit of mass.1 CSL = 1.4 times the mass of the
Sun. The smallest practical unit of time is Shake.
1 Shake = 10−8 s.

orders@surabooks.com Ph:9600175757 / 8124201000


This is Only for Sample
for Full Book Order Online and Available at All Leading Bookstores
Sura’s  XI Std - Physics ➠ Unit 01 ➠ Nature of Physical World and Measurement 27

61. Which of these unit is largest: AU, light year 67. What is dimensional equation? Give an example.
and parsec. Express the average distance of Ans. When the dimensional formula of a physical
earth from the sun in (i) light year (ii) per sec. quantity is expressed in the form of an equation,
Ans. Parsec is the largest unit. such equation is known as the dimensional equation.
parsec > light year > AU Ex: acceleration = [M0LT-2]

m
Average distance of earth from the sun is (d) 68. Define FPS system of units.
(astronomical unit). Ans. It is the British Engineering system of units, which
uses foot, pound and second as the basic units
(i) d = 1 AU = 1.496 × 1011 m
for measuring length, mass and time respectively.

co
1.496 × 1011 69. Define CGS system of units.
= 15
= 1.58 × 10 −5 light year.
9.46 × 10 Ans. It is the Gaussian system of units, which uses
1.496 × 1011 centimeter, gram and second as the basic units
(ii) d = par sec.
3.08 × 1016 for measuring length, mass and time respectively.

s.
= 4.86 × 10–6 par sec. 70. Define MKS system of units.
62. The radius of gold nucleus is 41.3 Fermi. Ans. It is the Metric system of units, which uses metre,
Express its volume in m3. kilogram and second as the basic units for measuring

ok
Ans. Radius of gold nucleus = 41.3 × 10–15 m
length, mass and time respectively.
4 3 4 71. Define one radian (S.I standard for plane angle)
Volume (V) = πr = × 3.14 × (41.3 × 10–15)3 Ans. It is the angle subtended at the centre of a circle
3 3
by an arc equal in length to the radius of the circle.
V = 2.95 × 10–40 m.
o 180º
1 rad = p = 57.27º.
63. Describe the relation of Physics with mathematics.
72. Define one steradian (S.I standard for solid
ab
Ans. (i) Physics is a quantitative science.
(ii) Physics is closely related to mathematics as angle)
Ans. It is the solid angle subtended at the centre of a
a tool for its development.
sphere, by that surface of the sphere, which is
64. What is the difference between Accuracy and equal in area to the square of radius of the sphere.
ur

Precision? III. Short Answer Questions :


Ans.
 3 Marks
S.No. Accuracy Precision
1. Measurements Measurements
.s

close to true value. close to each 1. Give any three applications of physics in our
other. society.
2. All the accuracy All the precised Ans. (i) Basic laws of electricity and magnetism led
w

values are values to the discovery of wireless communication


precised. are not accurate. technology which has shrunk the world with
effective communication over large distances.
65. Describe the Personal errors. (ii) The launching of satellite into space has
w

Ans. These errors occur due to individual performing revolutionized the concept of communication.
experiment without initial setting up or careless (iii) Microelectronics, lasers, computers,
observation without precautions. superconductivity and nuclear energy have
w

66. Describe the errors due to external causes. comprehensively changed the thinking and
Ans. These errors are due to external conditions like living style of human beings.
change in temperature, humidity or pressure during
an experiment.

orders@surabooks.com Ph:9600175757 / 8124201000


This is Only for Sample
for Full Book Order Online and Available at All Leading Bookstores
28 Sura’s  XI Std - Physics ➠ Volume - I ➠ Unit 01 ➠ Nature of Physical World and Measurement

2. What are fundamental quantities and derived 6. Give the values for the following units with
quantities? (Sep. 2020) prefixes.
Ans. Fundamental quantities (i) 1 Mega ohm (ii) 1 milliampere
Fundamental quantities are quantities which (iii) 1 deca gram (iv) 1 nano second
cannot be expressed in terms of any other (v) 1 micro volt (vi) 1 centimetre
physical quantity.

m
Ans.
Example: Quantities like length, mass, time, (i) 1 Mega ohm (MW) = 106 Ω
temperature are fundamental quantities.
(ii) 1 milliampere (mA) = 10–3A
Derived quantities:
(iii) 1 daca gram (da g) = 10g

co
Quantities that can be expressed in terms of
fundamental quantity are called derived quantities. (iv) 1 nano second (ns) = 10–9s
Example: Quantities like area, volume, velocity (v) 1 microvolt (mV) = 10–6V
are derived quantities. (vi) 1 centimetre (cm) = 10–2m
3. What are fundamental units and derived units?

s.
7. What are the advantages of the SI system?
Ans. Fundamental units:
Ans. (i) This system makes use of only one unit
The units in which the fundamental quantities are
for one physical quantity, which means a
measured are called fundamental units. It is also

ok
rational system of units.
known as base units.
Derived units: (ii) In this system, all the derived units
The units in which the derived quantities are can be easily obtained from basic and
measured are called Derived units. supplementary units, which means it is a
coherent system of units.
Example:
o
Unit of distance m (iii) It is a metric system which means that
Unit of speed = = s = ms–1
ab
Unit of time multiples and submultiples can be expressed
as powers of 10.
ms–1 is a derived unit.
4. Name the SI unit for electric current and give 8. Distinguish between fundamental and derived
a definition for it. units.
ur

Ans. The SI unit for electric current is ampere (A) Ans.


Definition: Fundamental Units Derived Units
One ampere is the constant current, which when 1. Using fundamental Using derived
maintained in each of the two straight parallel units fundamental units, derived
.s

conductors of infinite length and negligible cross quantities are quantities are
section, held one metre apart in vacuum shall measured measured
produce a force per unit length of 2 × 10−7 N/m 2. These units cannot These units can be
w

between them. be expressed in expressed in terms


terms of other of all fundamental
5. Name the SI unit for Luminous intensity and
fundamental units units.
give a definition for it.
w

3. Examples : metre, Examples: metre/


Ans. The SI unit for Luminous intensity is candela. Its
kilogram, second second, newton/
symbol is cd. (m, s, kg. A. mol) meter, kilogram/
Definition: metre seconds.
w

One candela is the luminous intensity in a given (m/s, kg/m3)


direction, of a source that emits monochromatic 9. Give any three practical units of time.
radiation of frequency 5.4 × 1014 Hz and that has
Ans. (i) Solar year :
1
a radiant intensity of watt/steradian in that It is the time taken by the earth to complete
638
direction. one revolution around the sun in its orbit.
1 solar year = 365.25 average solar days.

orders@surabooks.com Ph:9600175757 / 8124201000


This is Only for Sample
for Full Book Order Online and Available at All Leading Bookstores
Sura’s  XI Std - Physics ➠ Unit 01 ➠ Nature of Physical World and Measurement 29

(ii) Leap year: a1 + a2 + a3 + .............an


The year which is divisible by 4 and in am =
1 i=n n
which the month of February has 29 days am = ∑ ai
is called leap year. n i =1
(iii) Lunar month : Usually this arithmetic mean is taken as the
best possible true value of the quantity.

m
It is the time taken by the moon to complete
12. Explain unification with example.
one revolution around the earth in its orbit.
Ans. (i) Attempting to explain diverse physical phenomena
1 lunar month = 27.3 days.
with a few concepts and laws is unification.

co
10. What is Gross Error & How can it be
(ii) Newton’s universal law of gravitation explains
minimised? the motion of freely falling bodies towards the
Ans. Gross Error
Earth, motion of planets around the Sun, motion
(i) The error caused due to the shear carelessness of the Moon around the Earth, thus unifying the

s.
of an observer is called gross error. fundamental forces of nature.
For example 13. Explain reductionism with example.
(ii) Reading an instrument without setting it Ans. (i) An attempt to explain a macroscopic system

ok
properly. in terms of its microscopic constituents is
(iii) Taking observations in a wrong manner reductionism.
without bothering about the sources of (ii) For example, thermodynamics was developed to
errors and the precautions. explain macroscopic properties like temperature,
(iv) Recording wrong observations. These errors
o
can be minimized only when an observer is
entropy, etc., of bulk systems.
(iii) The above properties have been interpreted
careful and mentally alert.
ab
in terms of the molecular constituents
11. Explain Random errors.
(microscopic) of the bulk system by kinetic
Ans. Random errors
theory and statistical mechanics.
(i) Random errors may arise due to random and 14. How are theoretical predictions useful?
unpredictable variations in experimental Ans. (i) Theoretical predictions aided with recent
ur

conditions like pressure, temperature, voltage simulation and computation procedures are
supply etc. widely used to identify the most suited materials
(ii) Errors may also be due to personal errors by for robust applications.
the observer who performs the experiment.
.s

(ii) The pharmaceutical industry uses this technique


Random errors are sometimes called “chance very effectively to design new drugs. Bio
error”. compatible materials for organ replacement
(iii) When different readings are obtained by a
w

are predicted using quantum prescriptions of


person every time he repeats the experiment, physics before fabrication.
personal error occurs. (iii) Thus, experiments and theory work, hand in
(iv) For example, consider the case of the thickness
w

hand, complimenting one another.


of a wire measured using a screw gauge. 15. In what way physics is exciting us?
(v) The readings taken may be different for Ans. A small number of basic concepts and laws can
different trials. In this case, a large number of
w

explain diverse physical phenomena.


measurements are made and then the arithmetic (i) The most interesting part is the designing of
mean is taken. useful devices based on the physical laws.
(vi) If n number of trial readings are taken in For example i) use of robotics ii) journey to
an experiment, and the readings are a1, a2, Moon and to nearby planets with controls
a3,……………. an. The arithmetic mean is from the ground iii) technological advances
in health sciences etc.

orders@surabooks.com Ph:9600175757 / 8124201000


This is Only for Sample
for Full Book Order Online and Available at All Leading Bookstores
30 Sura’s  XI Std - Physics ➠ Volume - I ➠ Unit 01 ➠ Nature of Physical World and Measurement

(ii) Carrying out new challenging experiments to 19. How is physics useful in geology and
unfold the secrets of nature and in verifying oceanography?(or) Describe the relation of
or falsifying the existing theories. Physics with Geology.
(iii) Probing and understanding the science behind Ans. (i) Diffraction techniques helps to study the crystal
natural phenomena like the eclipse, and why structure of various rocks.

m
one feels the heat when there is a fire? (or) (ii) Radioactivity is used to estimate the age of
What causes the wind, etc. rocks, fossils and the age of the Earth.
16. Write the role of Physics in Technology. (iii) Oceanographers seek to understand the physical
Ans. (i) Basic laws of electricity and magnetism led and chemical processes of the oceans. They

co
to the discovery of wireless communication measure parameters such as temperature,
technology which has shrunk the world with salinity, current speed, gas fluxes, chemical
effective communication over large distances. components.
(ii) The launching of satellite into space has 20. How can the systematic errors be minimised?

s.
revolutionized the concept of communication. Ans. (i) By choosing the instrument carefully.
(iii) Microelectronics, lasers, computers, (ii) Necessary correction is to be made.
superconductivity and nuclear energy have (iii) High precision instrument is to be used.

ok
comprehensively changed the thinking and (iv) Proper setting up of experiments is to be
living style of human beings. done.
17. In what way Physics is in relation to Chemistry. (v) Taking proper precautions is a must, while
Ans. (i) In physics, we study the structure of atom, making observations.
o
radioactivity, X-ray diffraction etc. 21. What are systematic errors? (or) What are the
(ii) Such studies have enabled researchers in Classifications of Systematic errors?
ab
chemistry to arrange elements in the periodic Ans. Systematic error is reproducible inaccuracies that
table on the basis of their atomic numbers. are consistently in the same direction. They can
(iii) This has further helped to know the nature of be classified as
valence and chemical bonding and to understand (i) Instrumental errors,
the complex chemical structures. Inter-disciplinary (ii) Imperfection in experimental technique or
ur

branches like Physical chemistry and Quantum procedure


chemistry play important roles here. (iii) Personal errors.
18. What is the relation of Physics to Biology? (iv) Errors due to external causes
.s

Ans. (i) Biological studies are impossible without a (v) Least count error.
microscope designed using physics principles. 22. Describe the relation of Physics with Psychology.
(ii) The invention of the electron microscope has Ans. (i) All the psychological interactions can be
w

made it possible to see even the structure of a derived from a physical process.
cell. X-ray and neutron diffraction techniques (ii) The movements of neurotransmitters are
have helped us to understand the structure governed by the physical properties of diffusion
w

of nucleic acids, which help to control vital and molecular motion.


life processes. X-rays are used for diagnostic (iii) The function of our brain is related to our
purposes. underlying dualism (wave -particle nature).
23. Describe Instrumental errors. How is it minimised?
w

(iii) Radio-isotopes are used in radiotherapy for


the cure of cancer and other diseases. In recent Ans. (i) It is happened when an instrument is not
years, biological processes are being studied calibrated properly at the time of manufacture.
from the physics point of view. (ii) For example, If a measurement is made with
a meter scale whose end is worn out, result
obtains error.

orders@surabooks.com Ph:9600175757 / 8124201000


This is Only for Sample
for Full Book Order Online and Available at All Leading Bookstores
Sura’s  XI Std - Physics ➠ Unit 01 ➠ Nature of Physical World and Measurement 31

(iii) These errors can be rectified by using the 2. Explain propagation of errors in the difference
good quality instruments. of two quantities and also in the division of two
24. Write a note on parallax method. quantities.
Ans. (i) Parallax is the name given to the object with Ans. Errors in the difference of two quantities :
respect to the background, when the object Let ΔA and ΔB be the absolute errors in the two
is seen from two different positions. quantities, A and B, respectively. Then,

m
(ii) The distance between the two positions is Measured value of A = A ± ΔA
called basis (b). Measured value of B = B ± ΔB
(iii) This method is used for measuring very large Consider the difference, Z = A – B
The error ΔZ in Z is then given by

co
distance such as distance of a planet or star.
IV. Long Answer Questions : Z ± ΔZ = (A ± ΔA) – (B ± ΔB)
5 Marks = (A – B) ± (ΔA + ΔB)
 = Z ± (ΔA + ΔB)
1. Discuss the relation of physics with other branches (or) ΔZ = ΔA + ΔB

s.
of science The maximum error in difference of two
Ans. Physics is the most fundamental branch of science. quantities is equal to the sum of the absolute
It has played a key role in the development of all errors in the individual quantities.
branches of sciences. Error in the division or quotient of two

ok
Physics in relation to mathematics quantities
ÊÊ Physics is a quantitative science. Mathematics Let ΔA and ΔB be the absolute errors in the two
provides the necessary signs and tools which quantities A and B respectively.
the physicist use. A
Consider the quotient, Z =
o
ÊÊ It has played an important role in the B
development of theoretical physics. The error ΔZ in Z is given by
ab
ÊÊ Had newton not invented calculus, he would ∆A 
A 1 ± 
not have been able to discover the universal A ± ∆A  A 
law of gravitation. Z±Z = =  ∆B 
B ± ∆B B 1±
Physics in relation to chemistry:  
B
ÊÊ In physics, we study the structure of atom,
ur

−1
A  ∆A   ∆B 
radio activity, X-ray diffraction, etc. = 1±  1± 
B A  B
ÊÊ Such studies have enabled chemists to arrange
elements in the periodic table on the basis of  ∆A   ∆B 
their atomic numbers. or Z ± ∆Z = Z 1± A  1 B 
.s

ÊÊ This has further helped to know the nature [using (1+x)n ≈ 1 + nx, when x<<1]
of valency and chemical bonding and to
understand the complex chmical structures. Dividing both sides by Z, we get,
w

Physics in relation to biology: ∆Z  ∆A   ∆B 


1± = 1±  1 
ÊÊ The developments in life sciences a great Z A  B
deal to physics.
w

∆A ∆B. ∆A ∆B
ÊÊ Optical microscopes are extensively used in = 1±  ±
A B A B
the study of biology. As the terms ΔA/A and ΔB/B are small, their
ÊÊ With the help of an electron microscope, one product term can be neglected.
w

can study the structure of cell.


The maximum fractional error in Z is given by
ÊÊ The X-rays and neutron diffraction techniques
have helped in understanding the structure of ∆Z  ∆A ∆B 
=  + 
nucleic acids, which helped to control vital Z A B 
life process. The maximum fractional error in the quotient of
ÊÊ Radio isotopes are used in radiation therapy two quantities is equal to the sum of their individual
for the cure of deadly diseases like cancer. fractional errors.

orders@surabooks.com Ph:9600175757 / 8124201000


This is Only for Sample
for Full Book Order Online and Available at All Leading Bookstores
32 Sura’s  XI Std - Physics ➠ Volume - I ➠ Unit 01 ➠ Nature of Physical World and Measurement

3. Write to causes of errors in measurement. iv) If the digit to be i) 3.45 is rounded


Ans. dropped is 5 or 5 off to 3.4
(i) Least count error Associated with the followed by zeros, ii) 8.250 is
then the preceding rounded off to
poor resolution of digit is not changed 8.2.
the instrument if it is even

m
(ii) Instrumental Associated with the v) If the digit to be i) 3.35 is rounded
errors faulty calibration dropped is 5 or 5 off to 3.4
or change in followed by zeros, ii) 8.350 is
conditions. then the preceding rounded off to

co
(iii) Random errors Getting difficult digit is raised by 1 8.4
results for the same if it is odd
measurement done 5. Explain the rules framed to count significant
repeatedly figures with the examples.

s.
(iv) Personal errors Associated with the
Ans.
individual performing
the experiment ie. Rule Example
improper precautions, 1. All non-zero digits 1342 has four

ok
are significant significant figures
incorrect initial set up
2. All zeros between 2008 has four
of experiment. two non-zero digits significant figures
(v) Systematic errors Which tends to be in are significant
the same direction. 3. All zeros right 30700. has five
4.
o
Write the rules for rounding off. (or) Explain
to nonzero
digit but left to
significant figures
the rules framed for rounding off the numbers decimal point are
ab
with the examples. significant.
Ans. 4. The terminal or 30700 has three
trailing zeros in the significant figures
Rule Example number without
i) If the digit to be i) 7.32 is rounded decimal point are
ur

dropped is smaller off to 7.3 not significant.


than 5, then the ii) 8.94 is rounded 5. All zeros are 30700 m has five
preceding digit off to 8.9 significant if the significant figures.
should be left number given with
unchanged. measurement unit.
.s

ii) If the digit to be i) 17.26 is rounded 6. f a number is less i)0.00345


dropped is greater off to 17.3 than1, the zeros has three
than 5, then the 11.89 is rounded between decimal significant
point and first non- figures.
w

preceding digit ii) off to 11.9 zero digit are not


should be increased ii) 0.030400 has
significant but the
by 1 zeros right to last five significant
iii) If the digit to i) 7.352, on being non-zero digit figures.
w

be dropped is 5 rounded off to are significant. iii)


40.00 has four
followed by digits first decimal significant
other than zero, becomes 7.4 figures.
w

then the preceding ii) 18.159 on 7. The number of 1.53 cm, 0.0153 m,
digit should be being rounded significant figures 0.0000153 km all
raised by 1 off to first doesn’t depend on have three significant
decimal, the system of units figures.
become 18.2. used

orders@surabooks.com Ph:9600175757 / 8124201000


This is Only for Sample
for Full Book Order Online and Available at All Leading Bookstores
Sura’s  XI Std - Physics ➠ Unit 01 ➠ Nature of Physical World and Measurement 33

6. The speed of an object v = 40ms–1. The same


Numerical Problems quantity of speed in kmh–1 is,
 1 Mark (a) 60 (b) 160 (c) 40 (d) 144
 [Ans. (d) 144]
1. Find the value of one AU in 1000 km
Hint:

m
(a) 1.5 × 105m (b) 2.5 × 106m Quantity of speed in m/s = 40
18
(c) 1.5 × 1011m (d) 2.5 × 1010m Quantity of speed in km/h = 40 ×
5
 [Ans. (a) 1.5 × 105m] = 144 kmh–1.

co
Hint: 7. The speed of an object v = 90km/h. The same
1 AU = 1.5 × 1011m. quantity of speed in m/s is,
(a) 90 (b) 25 (c) 45 (d) 180
1.5 × 1011 m
1 AU in 1000 km =  [Ans. (b) 25]

s.
106 m
[ 1000km = 106m] Hint:
Quantity of speed in km/h = 90
= 1.5 × 105m 5

ok
Quantity of speed in m/s = 90 × = 25 m/s.
2. How many AU present in one light year? 18
(a) 6.30 × 104m (b) 9.46 × 1015m 8. 3.5kg mass of a metal plate has the volume of
1.5m3. Find the density of metal plate.
(c) 6.2 × 102m (d) 9.4 × 1016m
(a) 1.5 kg/m3 (b) 2.3 kg/m3

Hint:
o
[Ans. (a) 6.30 × 104m]
(c) 3.4 kg/m3 (d) 4.8 kg/m3

1 light year = 9.45 × 1015m.  [Ans. (b) 2.3 kg/m3]


ab
Hint:
\No. of AU in 1 light year
Density is a derived unit.
9.45 × 1015 m Mass 3.5kg
= = 6.30 × 104m
11
1.5 × 10 m Density = Volume = = 2.3 kg/m3
1.5m3
ur

3. How many mm present in one metre?


(a) 10–6mm (b) 106mm 9. The value of 1° is,
(c) 10–3mm (d) 10–2mm [Ans. (b) 106mm] (a) 1.745 × 10–2 rad (b) 1.946 × 10–11 rad
.s

Hint: (c) 3.6 rad (d) 3600 rad


1 micron = 1 µm = 10–6 m.  [Ans. (a) 1.745 × 10–2 rad]
Hint:
1
w

No. of µm in 1m = = 106 mm 2π π 22
10 −6
1° = = = = 1.745 × 10–2 rad
360 180 7 × 180
4. Express the derived unit of pressure.
w

Ans.
10. How many parsec are there in one kilometer?
Force
Pressure = Area (a) 3.084 × 10–16 (b) 3.08 × 108
unit of force N (c) 3.24 × 10–14 (d) None
w

= = 2 = Nm–2
unit of Area m Hint:  [Ans. (c) 3.24 × 10–14]
5. What is the formula representation of Mean 1 parsec = 3.08 × 1016m
Absolute error?
3.08 × 1016m = 1 parsec
1 n
Ans. Mean Absolute error, Dam = ∑ ∆ai 1 × 103
n i= 1 1km = 3.08 × 1016 parsec = 3.24 × 10–14 parsec

orders@surabooks.com Ph:9600175757 / 8124201000


This is Only for Sample
for Full Book Order Online and Available at All Leading Bookstores
34 Sura’s  XI Std - Physics ➠ Volume - I ➠ Unit 01 ➠ Nature of Physical World and Measurement

11. The angle of an object is 18.2°. What is the Area = length × breadth
angular diameter of the object in radians? SI unit of length =m
(a) 36.4 rad (b) 3.64 × 10–2rad SI unit of breadth Þ (length) = m
Area = length × breadth
(c) 31.74 × 10–2rad (d) 3.17 rad = m × m = m2.

m
Hint:  [Ans. (c) 31.74 × 10–2rad] 16. SI unit of the universal constant of gravitation
(G) is,
18.2 π 18.2 3.14 (a) kg–2 m–2 (b) kg ms–1
q = 18.2° = 60 × 60 × 180 = 60 × 60 × 180
(c) Nm2 kg–2 (d) Nm–1

co
= 31.74×10–2rad  [Ans. (c) Nm2 kg–2]
Hint: Universal constant of gravitation (G) is,
12. If a circle with 10 m radius and angle 60° at Fr 2 m1m2
centre, then what will be the length of arc? = mm  [F = G. 2 ]
r

s.
1 2
(a) 5.24m (b) 6.21m SI unit of force = N
(c) 7.1 mm (d) 10m [Ans. (a) 5.24m] SI unit of distance (r) = m = r2 = m2
SI unit of masses m1 and m2 = kg2

ok
Hint: π
Radius r = 10m, angle θ = 6 (60°) Fr 2 Nm 2
π Þ mm = = Nm2 kg–2.
kg 2
Length of the arc (l) = rq = 10 × 6 = 5.24m 1 2
o  2 Marks
13. The mass of an iron sheet is 0.250 kg and
1. Using a screw gauge the thickness of a wire
volume of the sheet is 1.5m3. Then what is the
was measured as 5mm. Calculate (i) the
ab
density of the iron sheet? Express the result in
SI unit system. fractional error (ii) the percentage error.
Given data:
(a) 0.267kg m–3 (b) 0.167 kg m–3
Thickness of the wire (t) = 5mm
(c) 0.255 kg m–3 (d) 0.285 kg m–3 Accurancy Dt = 0.01mm
ur

Hint:  [Ans. (b) 0.167 kg m–3] Solution:


Mass of the iron sheet = 0.250 kg ∆t 0.01
Volume of the iron sheet = 1.5 m3 (i) Fractional error = δt = = = 0.002.
t 5
.s

Mass 0.250
Density = = 1.5 = 0.167 kg m–3 ∆t
Volume (ii) Percentage error = δt = × 100%
14. What is the SI unit of linear momentum? t
= 0.002 × 100% = 0.02%
w

(a) ms–1 (b) ms–2


(c) kg ms–1 (d) kg m2s–1 2. If a mass of a proton is 1.67 × 10–27kg, how
many protons will be present in 1kg?
[Ans. (c) kg ms–1]
w


Hint:
Given data:
Linear momentum = mass × velocity
Mass of a proton = 1.67 × 10–27kg
SI unit of mass = kg
1.67 × 10–27kg is mass of 1 proton.
w

distance m
SI unit of velocity = = = ms–1 Solution:
time s
Linear momentum = kg ms–1. 1
1 kg is the mass of 1.67 × 10−27 protons
15. What is the SI unit of Area? = 0.5988 × 1027 protons
(a) m (b) m2 (c) Nm–1 (d) cm–1 = 5.988 × 1026 protons
Hint: 
[Ans. (b) m2] 5.988 × 1026 protons will be present in 1kg.

orders@surabooks.com Ph:9600175757 / 8124201000


This is Only for Sample
for Full Book Order Online and Available at All Leading Bookstores
Sura’s  XI Std - Physics ➠ Unit 01 ➠ Nature of Physical World and Measurement 35

3. Calculate angle of 1 second of arc. 7. The ratio of stress and strain of a wire is
Solution: 3 : 2. Find the co-efficient of elasticity. Express
1° the result in SI unit system.
1 second of arc = 1′′ = Solution:
60 × 60
The ratio of stress and strain of a wire is 3 : 2
1 π  1 3.14   π 

m
= × =  ×   1º = Force N
 SI unit of stress = = = Nm–2
60 × 60 180 3600 180  180  Area m 2
1 second of arc = 4.85 × 10–6 rad. Strain is dimensionless variable.
–15 Stress
The radius of a nucleus is 1.5 × 10 m of the

co
4. So, co-efficient of elasticity of a wire is = Strain
order fermi. Find the volume of the nucleus.
Solution:
3 Nm −2
Radius of a nucleus r = 1.5 × 10 –15
m. = = 1.5 Nm–2.
2 ( No unit )

s.
4
Volume of the nucleus V = 3 pr3.  3 Marks
1. In a following physical units, how many units

ok
4 4 are there in 1 metre?
= 3 × 3.14 × (1.5 × 10–15)3 = 3 × 3.14 × (4.5 × 10–45)
(i) 1 Astronomical unit (AU = 1.496 × 1011m
(ii) 1 light year = 9.467 × 1015m
= 18.84 × 10–45m3
(iii) 1 micron (m) = 10–6m
Volume of the nucleus is V = 18.84 × 10–45m3
5.
o
A beam of metal has length, breadth and height
(iv) 1 p a r a l l a c t i c s e c o n d ( p a r s e c )
= 3.08 × 1016m
as 4m, 3m and 5m respectively. Then what will
ab
Given data:
be the volume of the metal beam? Express the
result in SI unit system. 1 AU = 1.496 × 1011m
Solution: 1 ly = 9.467 ×1015m
Volume = length × breadth × height 1 mm = 10–6m
ur

SI unit of volume (v) = m × m × m = m3 1 parsec = 3.08 × 1016m


SI unit of length (l) = 4m Solution:
SI unit of breadth (b)= 3m (i) 1.496 × 1011m is equivalent to 1 AU.
.s

SI unit of height (h) = 5m 1


Volume (v) = l × b × h = 4 × 3 × 5 = 60m3 1 metre is equivalent to
1.496 × 1011
6. Find the SI unit of moment of inertia. 5.64kg
w

mass of a object is moving uniformly. The = 0.6684 × 10–11 = 6.68 × 10–12 AU


radius of gyration is measured as 30cm of an In one metre, 6.68 × 10–12 astronomical
object. Then what is the moment of Inertia? units are present
w

Solution: (ii) 9.467 × 1015m is equivalent to 1 ly.


Mass = 5.64kg 1
1 metre is equivalent to
w

radius of gyration = 30cm = 0.3m 9.467 × 1015


SI unit of a mass is kg = 0.1056 × 10–15 = 1.05 × 10–16 ly
SI unit of the radius (gyration) = m2 = 0.9
In one metre, 1.05 × 10–16 light years are
Moment of inertia (I) = mass × radius of gyration present.
= 5.64kg × 0.9m = 1.5228 kgm2

orders@surabooks.com Ph:9600175757 / 8124201000


This is Only for Sample
for Full Book Order Online and Available at All Leading Bookstores
36 Sura’s  XI Std - Physics ➠ Volume - I ➠ Unit 01 ➠ Nature of Physical World and Measurement

(iii) 10–6m equivalent of 1mm Solution:


1 1Å = 100 pm Þ 30pm = 0.3Å
1 metre is equivalent to −6 = 106mm
10 pm ® picometer (10–12m)
In one metre 106 microns are present Å = Angstrom (10–10)m
(iv) 3.08 × 1016 m equivalent to 1 parsec. Radius of the helium atom (r) = 30pm = 0.3Å

m
1 = 0.3 × 10–10m
1metre is equivalent to
3.08 × 1016 Volume of helium nucleus
= 0.324 × 10–16 = 3.24 × 10–17m 4
× 3.14 × (0.3 × 10–10)m3 = 1.256 × 10–30m3

co
V=
In one metre 3.24 × 10–17 parsec are present. 3
Number of atoms in 1 mole of helium atom
2. How many parallactic second are there in one
= Avogadro's number(N) = 6.023 × 1023
Astronomical unit?
Atomic volume of 1 mole of helium atoms (v΄)

s.
Given data:
V΄ = V × N
1 parallactic second = 3.08 × 1016 m V΄ = V × n
1 Astronomical unit = 1.496 × 1011 m = 1.256 × 10–30 × 6.023 × 1023

ok
Solution: = 7.564 × 10–7m.
1 AU 5. The radius of the platinum atom in a nucleus
1.496 × 1011 1.496 × 1011 × 10 −16
= = is 60.2 fermi. Find the volume of the nucleus.
1 parsec 3.08 × 1016 3.08
o Why fermi is used to measure size of a nucleus?
1.496 × 10 −5 Solution:
= = 0.485 × 10–5
3.08 The radius of the nucleus = 60.2 fermi
= 4.85 × 10–6 par sec.
ab

4.85 × 10–6 parsec present in one astronomical = 60.2 × 10–15m
unit. Volume of the platinum nucleus
3. If mass of an electron is 9.11 × 10–31 kg, how 4 4
= 3 pr3m3 = 3 × 3.14 × (60.2 × 10–15)3 m3
many electrons would weigh in 1 mg?
ur

Given data: 4
Mass of an electron = 9.11 × 10 kg –31 = 3 × 3.14 × (218167 × 10–45)
9.11 × 10–31kg is the mass of 1 electron 4
.s

= 3 × 3.14 × (2.18167 × 10–40)


Solution:
9.11 × 10–31kg = 9.11 × 10–31 × 103 g = 9.133 × 10–40m3
= 9.11 × 10–28 g
w

Vo l u m e o f t h e p l a t i n u m n u c l e u s i s
= 9.11 × 10–25 mg 9.133 × 10–40m3.
1 fermi is used for measure very small distance so
1mg is the mass of electron
w

9.11 × 10 −25 mg it is used to measure the nuclear size of an atom.


It is denoted as 1 Fermi = 10–15m.
= 0.1097 × 1025electrons = 1.097 × 1024 electrons
w

= 1.097 × 1024 electrons would weigh in 1 mg.


4. The unit of length convenient on the atomic
scale is known as angstrom and is denoted
by Å. The size of a helium atom is about
30 pico meter. What is the total atomic volume
in metre3 of one mole of helium atom?

orders@surabooks.com Ph:9600175757 / 8124201000


This is Only for Sample
for Full Book Order Online and Available at All Leading Bookstores
Sura’s  XI Std - Physics ➠ Unit 01 ➠ Nature of Physical World and Measurement 37

3. When the planet Jupiter is at a distance of


Value Based Questions 824.7 million kilometers from the earth, its
1. (i) Monica was watching the night sky. She saw angular diameter is measured to be 35.72 of
arc. Calculate the diameter of Jupiter.
a star, moving towards her, with increase in
brightness. After some few minutes when she Solution:
watched closely, she found it was the light Distance, D = 824.7 × 106 km

m
from a flight in the sky. She was surprised, 35.72 π
but initially the flight looked stationary, q = 35.72" = 60 × 60 × 180 rad
after some time it was glowing brightly q ® is angular diameter

co
moving towards her. So she went and asked
Diameter, d = ? ; d = D × q
her father. Why this effect occured?
35.72 π
(ii) Is RADAR used in launching a missile from = 824.7 × 106 × 60 × 60 × 180 km
the ground to hit the target (i.e. fight air
craft)? 35.72 π
= 824.7 × 106 × km

s.
×
Ans. (i) Any moving object, which is perpendicular 3600 180
to our eye sight will look like a stationary = 824.7 × 106 × 35.72 × 3.14 km
thing for a while. But as the moving object 3600 × 180

ok
changes its angle of vision, its movement will Diameter of Jupiter = 1.427 × 105 km
be known. 4. In a submarine fitted with a SONAR, the
(ii) Yes, RADAR is used in launching missile time delay between generation of a signal and
from the ground. o reception of its echo from an enemy ship is
110.3 seconds. If speed of sound in water is
Creative Questions (HOTS) 1450 ms–1 then calculate the distance of the
ab
1. Why has 'second' been defined is terms of periods enemy ship from the submarine.
of radiations from cesium-133? Solution:
Ans. Second has been defined is terms of periods of Speed of a sound in water 1450 ms–1
radiations because Time delay T = 110.3s
ur

(i) This period is accurately defined


vT
(ii) This period is not affected by change of Distance of the enemy ship, D = 2
physical conditions like temperature. pressure
and volume etc. 1450 × 110.3 159935
.s

(iii) The unit is easily reproducible in any good = 2


= 2
= 79967 m
laboratory.
Distance of enemy ship = 79.96 km
2. How many Astronomical units are there in
5. In an ocean surveillance system of ship fitted
one light year?
w

with a (RADAR), the time delay between


Given data:
generation of a radio waves reflected from an
1 Astronomical unit = 1.496 × 1011m enemy ship is observed to be 5.6s. Calculate
w

1 light year = 9.46 × 1015m the distance of the enemy ship from the
Solution:
surveillance ship.
Solution:
w

1 ly 9.46 × 10 15
15 −11 Time delay T = 5.6s
= = 9.46 × 10 × 10
1 AU 1.496 × 10 11 T 5.6
1.496 t = 2 = 2 = 2.8
9.46 × 104
= = 6.32 × 104AU Speed of radiowaves = speed of light (v)
1.496

In 6.32 × 104AU are present in one light year. = 3 × 108ms–1

orders@surabooks.com Ph:9600175757 / 8124201000


This is Only for Sample
for Full Book Order Online and Available at All Leading Bookstores
38 Sura’s  XI Std - Physics ➠ Volume - I ➠ Unit 01 ➠ Nature of Physical World and Measurement

Distance from the surveillance ship to enemy Least count of vernier calipers
ship D = v × t = 3 × 108 × 2.8 = 8.4 × 108m
= 1MSD – 1VSD = (1– 19 20 ) MSD
Distance (D) = 8.4 × 108 km.
1
= = 0.05cm.
Conceptual Questions 20

m
So screw gauge is more precise than vernier.
1. Why is it convenient to express the distance of
stars in terms of light year (or) parsec rather 3. If humans were to settle on other planets,
than in km? which of the fundamental quantities will be

co
in trouble? Why?
Ans. The distances of astronomical objects like stars,
Ans. Time will be in trouble. Time becomes irrelevant.
planets etc from the earth are huge. The distance
on the earth are relatively small so it can be Because day and year based on spinning and
measured in km. revolution of the planet. So each planet has its

s.
own year length.
For Example :
Eg. : Uranus and Neptune move too slow.
The distance to be next nearest big galaxy
Andromeda is 21,000,000,000,000,000,000 km. 4. Having all units in atomic standards is more

ok
useful. Explain.
i.e. 21 × 1018 km.
Ans. All units in atomic standards are more useful
This number is so large that it becomes hard to
because they never change with time.
write and to interpret.
5. Why dimensional methods are applicable only
So astronomical units like light year, parsec A.U
are used for large distances.
o up to three quantities?
Ans. If a quantity depends on more than three factors
2. Show that a screw gauge of pitch 1 mm and
ab
than dimensional formula cannot be derived.
100 divisions is more precise than a vernier
Because on equating the powers of M, L & T
caliper with 20 divisions on the sliding scale.
on either side of the dimensional equation, three
Ans. Least count of screw gauge
equations can be obtained, from which only three
ur

Pitch unknown dimensions can be calculated.


1
= No. of divisions = = 0.01 mm (or) 0.001cm
100
on the Head scale.
.s
w


w
w

orders@surabooks.com Ph:9600175757 / 8124201000

You might also like